Paeds SBAs Flashcards

1
Q

The paediatric team is resuscitating a 3-month-old boy who is in Pulseless Electrical Activity (PEA). He was discovered to be blue and lifeless when his parents went to wake him in the morning. The airway has been secured and despite ventilation the child remains blue. An intra-osseous needle has been inserted and two boluses of normal saline have been given. The nurse reports that the temperature of the child is 35°C. You listen to the chest and can hear bilateral breath sounds.

Where is the most appropriate position on the chest to do cardiac compressions?

Midline between the nipples
1 finger breadth above the line between the nipples
1 finger breadth below the line between the nipples
None of the above

A

1 finger breadth below the line between the nipples

In an infant the heart is lower in relation to the external landmarks than in older children or adults. The area of compression over the sternum should be one fingerbreadth below an imaginary line between the nipples.

How well did you know this?
1
Not at all
2
3
4
5
Perfectly
2
Q

You are in the acute assessment unit and see David, a 15-month-old boy who has a fever of 38.5°C. He has had a runny nose, cough and a fever for 3 days. Since this morning he has slept and has been difficult to wake. His heart rate is raised. He has a rash scattered over his legs which does not disappear with pressure.

Which of the following is the most likely diagnosis?

NAI
HSP
ITP
ALL
Septicaemia
A

Septicaemia

He has a purpuric rash, with lesions of variable size. In a febrile child, meningococcal septicaemia is most likely. This may be accompanied by meningitis.

Henoch–Schönlein purpura although localized to the legs, is associated with abdominal pain and joint pain but not with fever.

With Idiopathic thrombocytopenia the children are usually well.

This is a relatively short history of the child being unwell. In acute lymphoblastic leukaemia you would expect a longer history.

How well did you know this?
1
Not at all
2
3
4
5
Perfectly
3
Q

A 3-year-old boy who is unconscious arrives in Accident and Emergency. You manage his Airway, Breathing and Circulation. His blood glucose is normal. On examination you note that he has bilateral pinpoint pupils. His temperature and other vital signs are otherwise normal.

What is the most likely cause?

3rd nerve lesion
Severe hypoxia
Hypothermia
Tentorial herniation
Opiate poisoning
A

Opiate poisoning

Bilateral, pin-point pupils with coma can be caused by a pontine lesion or opiate poisoning. Opiate poisoning may occur in homes with illicit substance abusers or adults on methadone.

Third nerve lesions and tentorial herniation would cause a unilaterally dilated pupil.

Severe hypoxia would cause dilated pupils.

With hypothermia the child’s temperature would be low and causes dilated pupils.

How well did you know this?
1
Not at all
2
3
4
5
Perfectly
4
Q

You are called to see a 3-year-old boy with a high fever. The nurse is worried that he is very sleepy. As you walk into the resuscitation room he makes no spontaneous response. You try calling his name but he makes no response. On stimulation, his eyes open, he moans and he raises his hand and pushes your hand away.

What is this child’s Glasgow Coma Score?

8
9
10
11
12
A

9

Eyes: React to pain - 2
Verbal: Moans to pain - 2
Motor: Localises pain - 5

How well did you know this?
1
Not at all
2
3
4
5
Perfectly
5
Q

Ryan, aged 10 months, is rushed to the Accident and Emergency department after being found submerged in the bath. His mother runs screaming into the department saying ‘help my baby, please’.

Which is the next most appropriate step?

Commence bag and mask ventilation
In managing his airway, his head should be in the neutral position
Commence chest compressions in a ratio of 15:2
Assess patient, call for help
Remove wet clothing/towels

A

Assess patient, call for help

All resuscitation algorithms ensure that patient is assessed in a sequential manner, first initial assessment and then management in an A,B,C approach. Calling for help early on in these situations is paramount as you need many people for resuscitation. This baby is going to need his airway managing in the neutral position, with bag and mask ventilation and cardiac compressions in a ratio of 15:2; the wet clothing will also need removing.

How well did you know this?
1
Not at all
2
3
4
5
Perfectly
6
Q

Mohammed, aged 8 months, has been vomiting and off his feeds for two days. Initially he had episodes of crying uncontrollably, drawing his legs up into his abdomen as if in pain, and appeared fractious. His mother gave him some oral rehydration solution, but his vomiting continued and he has become lethargic. On admission to hospital he is in shock.

What is the most likely diagnosis?

Gastroenteritis
Malrotation
Strangulated hernia
Intussusception
Meckel diverticulum
A

Intussusception

Intussusception is the most likely cause of the pain and shock. Although this could be a strangulated hernia, this should be evident on clinical examination.

How well did you know this?
1
Not at all
2
3
4
5
Perfectly
7
Q

Mohammed, aged 8 months, has been vomiting and off his feeds for two days. Initially he had episodes of crying uncontrollably, drawing his legs up into his abdomen as if in pain, and appeared fractious. His mother gave him some oral rehydration solution, but his vomiting continued and he has become lethargic. On admission to hospital he is in shock. Intussusception is suspected.

Mohammed is 8 months old and weighs 8 kg. He needs a bolus of normal saline 0.9% to treat his shock.

What volume of fluid would you give initially?

40ml
160ml
320ml
680ml
800ml
A

160ml

This is 20 ml/kg initially, repeated as necessary.

How well did you know this?
1
Not at all
2
3
4
5
Perfectly
8
Q

Mohammed, aged 8 months, has been vomiting and off his feeds for two days. Initially he had episodes of crying uncontrollably, drawing his legs up into his abdomen as if in pain, and appeared fractious. His mother gave him some oral rehydration solution, but his vomiting continued and he has become lethargic. On admission to hospital he is in shock. Intussusception is suspected.

Mohammed has received the fluid bolus of normal saline 0.9% which has improved his condition. From his presentation you suspect he is 10% dehydrated. You receive his laboratory results which reveal a plasma sodium of 138 mmol/L (within the normal range). His continuing fluid loss from vomiting is small and can be ignored. (The maintenance intravenous fluid requirement for a child of this age is 100 ml/kg/24 h.)

What is Mohammed’s total fluid requirement for the initial 24 hours? He weighs 8 kg.

160ml
320ml
800ml
880ml
1600ml
A

1600ml

Mohammed’s fluid requirement is calculated by adding:
• Deficit: 10% of 8 kg = 800 ml
• Maintenance: 100 ml/kg/24 h = 800 ml
• Continuing losses: 0 ml
Total = 1600 ml
How well did you know this?
1
Not at all
2
3
4
5
Perfectly
9
Q

You are called to the resuscitation room where there is a 6-year-old child who has arrived by ambulance. He is a known to have epilepsy and is on anti-epileptic treatment. The child has been having a generalised seizure for 15 minutes. The ambulance crew gave a dose of buccal midazolam 5 minutes ago. The emergency doctor has maintained the airway and has applied oxygen with a non-rebreathe mask. His capillary refill time is less than 2 seconds and his heart rate 120 beats/minute.

What is the next most appropriate management step?

Administer further anti-convulsant
Gain iv access
Check blood glucose level
Check pupils
Check conscious level
A

Check blood glucose level

This is the most appropriate next step as, if the patient is hypoglycaemic, the only treatment to stop the fit would be to administer glucose.

How well did you know this?
1
Not at all
2
3
4
5
Perfectly
10
Q

Seb, a 2-year-old boy, was at his cousin’s birthday party. His mother noticed that he has suddenly developed a widespread urticarial rash and has also become flushed in the face. His vital signs are normal and he has no respiratory compromise.

Which medication would you give?

im adrenaline
Oral corticosteroid
iv hydrocortisone
oral antihistamine
im antihistamine
A

Oral antihistamine

In children, the most common causes of acute food allergy are ingestion or contact with nuts, egg, milk or seafood. Urticaria and facial swelling are mild reactions. Immediate management is with an oral antihistamine (e.g. chlorpheniramine) and observed over 2 hours for possible

How well did you know this?
1
Not at all
2
3
4
5
Perfectly
11
Q

Jenny, a 3-year-old girl, was at a village fete. She suddenly developed swollen cheeks and lips and a widespread urticarial rash. She is rushed to the nearby general practice surgery, where it is noted that her breathing is very noisy. She is distressed and frightened. On auscultation she has widespread wheeze.

Which medication would you give first?

im adrenaline
Oral corticosteroid
iv hydrocortisone
oral antihistamine
im antihistamine
A

IM adrenaline

This child has anaphylaxis, which is life-threatening as she has upper airway obstruction and bronchoconstriction. Priority is to manage the airway and give oxygen via a non-rebreathe mask. The first medication to give would be intramuscular adrenaline.iv hydrocortisone should only be given after immediate treatment of the upper airway obstruction with intramuscular adrenaline. Also, her upper airway obstruction may be further compromised by the distress of establishing an intravenous cannula. It takes about 6 hours to have optimal effect.

This child is unlikely to be able to take oral medications and the antihistamine alone will not treat the upper airway obstruction and oral steroids would take too long to work.

im antihistamine would make the situation worse, as it is painful and will not directly treat the upper airway obstruction.

How well did you know this?
1
Not at all
2
3
4
5
Perfectly
12
Q

There has been a dramatic decline in the incidence of sudden infant death syndrome in the UK.

Which of the following is the single most important factor responsible for this decline?

Feet to foot of cot
Supine sleeping
Keeping baby in parent’s room until 6 months
Keeping room cool
Parents should not smoke in same room as infant

A

Supine sleeping

All the answers have helped reduce the risk of cot death, but the single most important factor is putting babies to sleep on their backs.

How well did you know this?
1
Not at all
2
3
4
5
Perfectly
13
Q

What is the most common cause of death in children aged 1–14 years in the UK?

Accidents
Congenital heart disease
Infectious diseases
Malignant disease
Respiratory disease
A

Accidents

This is now the commonest cause of death, followed by malignant disease.

How well did you know this?
1
Not at all
2
3
4
5
Perfectly
14
Q

Hamim, a boy aged 3 years, fell 3 metres from a first-floor balcony on to a concrete path. He presents to the Accident and Emergency department with his parents who are concerned that he has vomited several times since the episode. After the fall he immediately cried out in pain, but appeared to be all right. His mother reports that he did not lose consciousness. On examination he is found to be fully conscious but has a large bruise over the left parietal region. There are neither focal neurological signs nor any other injuries. His heart rate is 110 beats/minute, his respiratory rate is 25 breaths/minute and his blood pressure is 90/50 mmHg.

Which of the following would be the most serious clinical sign?

Nasal discharge since his fall
Further enlargement of parietal bruise
A fractured nose
Laceration above the eye requiring suturing
A unilateral black eye
A

Nasal discharge since his fall

This child has had a significant head injury. His vital signs are normal. A nasal discharge post head trauma is a significant sign. It may be leakage of cerebrospinal fluid (CSF) that can indicate a basal skull fracture.

Parietal bruising, a fractured nose or facial laceration are all distressing, but do not suggest significant brain or skull injury.

Black eyes (periorbital ecchymosis) are a significant sign if bilateral, as this can indicate a basal skull fracture.

How well did you know this?
1
Not at all
2
3
4
5
Perfectly
15
Q

Hamim, a boy aged 3 years, fell 3 metres from a first-floor balcony on to a concrete path. He presents to the Accident and Emergency department with his parents who are concerned that he has vomited several times since the episode. After the fall he immediately cried out in pain, but appeared to be all right. His mother reports that he did not lose consciousness. On examination he is found to be fully conscious but has a large bruise over the left parietal region. There are neither focal neurological signs nor any other injuries. His heart rate is 110 beats/minute, his respiratory rate is 25 breaths/minute and his blood pressure is 90/50 mmHg.

Eight hours after admission, the nurses note a change in his level of consciousness. He is now responsive only to painful stimuli; his left pupil is dilated although still responsive to light. His airway, breathing and circulation are satisfactory. A CT scan shows that there is a haemorrhage and a skull fracture. He is stabilized in the resuscitation room.

Which of the following is the most appropriate next step in his management?

Clotting studies
EEG
Skeletal survey
Ophthalmology opinion
Neurosurgical referral
A

Neuro referral

This child has sustained a potentially serious head injury and now has reduced level of consciousness and focal neurological signs, which are indications to be assessed by a neurosurgical specialist. The priority for this child is to prevent further secondary brain injury.

How well did you know this?
1
Not at all
2
3
4
5
Perfectly
16
Q

Chelsea is a 2-year-old girl who presented 6 months ago with a fractured femur which was felt to be accidental. She presents to Accident and Emergency having slipped in the bath whilst briefly being left alone. On examination there is swelling and bruising over Chelsea’s anterior right chest wall. She has some older bruises on her right thigh. She has no other medical problems and is not on any medication. The chest X-ray reveals rib fractures.

Select the most appropriate next management:

Genetic counselling
Parental reassurance
Child protection case conference
Discharge home without follow up
Health visitor home assessment
A

Child protection case conference

Even if you do not see the fractures on this X-ray, there are some features in this history which are very concerning. The child is left unsupervised in the bath; she has had a previous femur fracture and has bruises on her thigh. A child protection conference needs to be undertaken to assess the safest and most appropriate outcome for this child.

How well did you know this?
1
Not at all
2
3
4
5
Perfectly
17
Q

You are a junior doctor working on the paediatric ward. You are asked to take some bloods from Chloe, an 11-year-old girl. Her parents do not wish to be present. When rolling up her sleeve to look for a suitable place for venepuncture you note numerous bruises from strap marks to her upper arm. You ask her how she got the bruising. She replies that her uncle did it as she had been misbehaving.

What should you do with the information?

Ignore it; she was being disciplined for misbehaviour
Just document what was said in the notes including sketches and photographs
Document what was said in the medical notes including sketches and photographs and inform the Consultant on call.
Inform the health visitor and request a home assessment
Inform her mother what she said and suggest she asks the uncle about it

A

Document what was said in the medical notes including sketches and photographs and inform the Consultant on call.

This is a safeguarding issue, as the girl has alleged physical abuse and this needs to be taken seriously. Other agencies e.g. social services and school need to be contacted to identify any concerns, and the patient needs a full medical examination by a paediatrician trained in child protection.

How well did you know this?
1
Not at all
2
3
4
5
Perfectly
18
Q

Louise, aged 4 years, was hit by a car in the local supermarket car park. She is brought to the Accident and Emergency department by the paramedical team.

An initial assessment shows:
• Airway —neck collar in place, talking to mother
• Breathing —receiving oxygen via a rebreathing circuit, oxygen saturation 99%
• Breathing —air entry satisfactory and equal bilaterally, respiratory rate 30/min
• Circulation —pulse 160/minute, blood pressure 90/50, capillary refill time 3 sec
• Disability —alert, but frightened and agitated, moving all four limbs.

She has abrasions to her left flank and pain in her left shoulder.

What is the next intervention needed?

Intubation and ventilation
IV access
CXR
Analgesia
Blood glucose
A

IV access

Louise has cardiovascular compromise and urgently needs fluid resuscitation. She therefore needs intravenous access.

A chest X-ray is needed but this comes after ABC.

Analgesia and a blood sugar are important but not until she has some fluid.

How well did you know this?
1
Not at all
2
3
4
5
Perfectly
19
Q

Louise, a girl aged 4 years, was hit by a car in the local supermarket car park. She is brought to the Accident and Emergency department by the paramedical team.

An initial assessment shows:
• Airway —neck collar in place, talking to mother
• Breathing —receiving oxygen via a rebreathing circuit, oxygen saturation 99%
• Breathing —air entry satisfactory and equal bilaterally, respiratory rate 30/min
• Circulation —pulse 160/min, blood pressure 90/50, capillary refill time 3 sec
• Disability —alert, but frightened and agitated, moving all four limbs.

She has abrasions to her left flank and pain in her left shoulder.

Chest and abdominal X-rays show fractures of the 9th and 10th ribs on the left-hand side.

What is the most important investigation to perform to establish the cause of her condition?

Abdo US
CT head
Cervical spine X-ray
FBC
Blood creatinine, U&Es
A

Abdo US

The history and signs suggest hypovolaemic shock from splenic injury. She needs an urgent abdominal ultrasound scan (FAST scan, focused abdominal sonography in trauma).

How well did you know this?
1
Not at all
2
3
4
5
Perfectly
20
Q

Ronaldo is a 2½-year-old boy from Brazil. He pulled a chip pan off the cooker and has been extensively burnt. He is rushed to the nearest Accident and Emergency department. His airway, breathing and circulation are satisfactory. His burns are to his chest, abdomen and his right arm and hand. Most of the burnt area is now blistering and mottled in colour, with a few white areas. Intravenous analgesia is given.

What immediate management does Ronaldo require?

IV 0.9% saline
Place in cold water
Cover the burns with sterile dressings
Intubation and artificial ventilation
IV antibiotics
A

IV 0.9% saline

There will be significant fluid loss through the burnt areas, which needs replacing.

How well did you know this?
1
Not at all
2
3
4
5
Perfectly
21
Q

Jake is a 3-year-old boy. His brother spilt a pan of hot water over him and he has been extensively burnt. He is rushed to the nearest Accident and Emergency department. His airway and breathing are satisfactory. He is tachycardiac, his capillary refill time is 4 seconds and he has a low blood pressure. He has 20% burns involving his chest, abdomen and his right arm and hand.

What is the most important underlying cause for his tachycardia and a capillary refill of 4 seconds?

Shock secondary to his pain from his burns
Shock due to loss of blood plasma, because of damage to his blood vessel secondary to his burn
He has shock due to loss of red blood cells, because of damage to his blood vessels secondary to his burn
He has shock due to vasodilation of his blood vessels, secondary to his burn
He has shock due to vasodilation of his blood vessels, secondary to infection developing in his burn

A

Shock due to loss of blood plasma, because of damage to his blood vessel secondary to his burn

Jake has hypovolaemic shock secondary to the loss of blood plasma. This is secondary to loss of skin integrity.

How well did you know this?
1
Not at all
2
3
4
5
Perfectly
22
Q

Solomon, aged 3 years, has been found eating some of his pregnant mother’s iron tablets; up to 10 tablets are missing. Their general practitioner advised that he should be taken to hospital directly. On examination in the emergency department he is found to be talkative, with no obvious abnormalities. He has no other medical problems and is not normally on any medications.

What would be the first investigation you would perform?

Abdo Xray
FBC
Clotting
Serum iron
LFTs
A

Abdo Xray

An abdominal X-ray identifies if there is a significant number of tablets in his stomach.

The serum iron result will not be helpful at this stage as he will not have absorbed the medication.

How well did you know this?
1
Not at all
2
3
4
5
Perfectly
23
Q

Solomon, aged 3 years, has been found eating some of his pregnant mother’s iron tablets; up to 10 tablets are missing. Their general practitioner advised that he should be taken to hospital directly. On examination in the emergency department he is found to be talkative, with no obvious abnormalities. He has no other medical problems and is not normally on any medications.

Your investigation suggests a significant ingestion of iron.

Which of the following would you initiate?

IV acetylcusteine
IV naloxone
IV desferrioxamine
Forced alkaline diuresis
Intubate and hyperventilate
A

IV desferrioxamine

Intravenous desferrioxamine binds with iron in the blood excreting it in urine and faeces.

How well did you know this?
1
Not at all
2
3
4
5
Perfectly
24
Q

Pauline is a 6-year-old girl. Her teacher is concerned, as she has been rubbing herself ‘down below’ in the classroom and touching other girls. She later discloses to her teacher that her stepfather has hurt her with his ‘willy’. She is seen by the consultant paediatrician who notices some vulval soreness and so takes a swab which reveals gonococcus. She also notices that there is some bruising to the thighs. She plots her weight and finds it to be just above the 99th centile. She has no other medical problems.

Which of the following findings is the most suggestive of sexual abuse?

Bruising to the thighs
Disclosure of event to teacher
Gonococcus on swab
Sexualised behaviour
Vulval soreness
A

Gonococcus on swab

All of these could suggest sexual abuse but only the gonococcus on the swab can absolutely confirm it. Identifying a sexually transmitted infection in a child is highly suggestive of sexual abuse. It will not however inform you of whom the perpetrator is.

The disclosure is very useful and a police investigation would need to be undertaken. But only the gonococcus on the swab can absolutely confirm sexual abuse.

Bruising over the thighs is common in active children but it is more concerning if it is found in the inner thigh as this area is anatomically ‘protected’ and not often bruised accidentally. But only the gonococcus on the swab can absolutely confirm sexual abuse.

How well did you know this?
1
Not at all
2
3
4
5
Perfectly
25
Q

Mr and Mrs Walsh attend clinic with their new baby, Ophelia, who has Down syndrome. They are keen to have further children and want to know more about their future risk of having children with Down syndrome. What chromosomal abnormality is likely to have caused Ophelia to have Down syndrome?

Non-disjunction
Translocation
Mosaicism
Point mutation
Triplet repeat expansion
A

Non-disjunction

It is responsible for 94% of cases of Down syndrome. the pair of chromosome 21s fails to separate, so that one gamete has two chromosome 21s and one has none. related to maternal age.

Translocation (5%) - When the extra chromosome 21 is joined onto another chromosome (usually chromosome 14, but occasionally chromosome 15, 22 or 21), this is known as a Robertsonian translocation.

Mosaicism (1%) - In mosaicism, some of the cells are normal and some have trisomy 21. This usually arises after the formation of the chromosomally normal zygote by non-disjunction at mitosis but can arise by later mitotic non-disjunction in a trisomy 21 conception. The phenotype is sometimes milder in Down syndrome mosaicism.

How well did you know this?
1
Not at all
2
3
4
5
Perfectly
26
Q

Mr and Mrs David are seen by the geneticists as their baby, Sarah, has Down syndrome. Her chromosomes are examined. Three copies of chromosome 21 are seen, one of which is attached to chromosome 14. How would you describe this abnormality?

Non-disjunction
Balanced Robertsonian translocation
Unbalanced Robertsonian translocation
Mosaicism
Triplet repeat expansion
A

Unbalanced translocation.

This is important because there is an increased risk of recurrence as one of the parents is likely to have a balanced translocation whereby one of their copies of chromosome 21 is attached to chromosome 14.

How well did you know this?
1
Not at all
2
3
4
5
Perfectly
27
Q

Olive is a 10-day-old baby with Down syndrome. On examination, you hear a loud heart murmur. What is the most likely cause?

Patent ductus arteriosus
Aortic stenosis
Coarctation of the aorta
Innocent murmur
Atrioventricular septal defect
A

Atrioventricular septal defect.

This is the most common congenital cardiac anomaly in children with Down syndrome. 40% of children with Down syndrome have a congenital heart defect and therefore it is important that all children have an echocardiogram in the neonatal period.

How well did you know this?
1
Not at all
2
3
4
5
Perfectly
28
Q

Fiona is a well 4-year-old girl with Down syndrome. She attends her yearly follow-up appointment with her mother. There are no real problems other than constipation, for which her general practitioner has started treatment. When you plot Fiona on the Down syndrome growth chart, you notice that her height is starting to drift from the 75th to the 25th centile. Her weight, however, has gone from the 50th to the 75th centile.

Which of the following investigations would you perform?

FBC
TFT
Cytogenetic testing
Coeliac screen
U&Es
A

Thyroid function tests

Children with Down syndrome are at an increased risk of hypothyroidism. Growth failure and constipation are symptoms of hypothyroidism. Coeliac disease is more common in Down syndrome and it can cause short stature; however her weight would be falling.

How well did you know this?
1
Not at all
2
3
4
5
Perfectly
29
Q

Mary is an infant with Down syndrome. She was diagnosed soon after birth and this has been confirmed with a rapid karyotype. Her antenatal scans had been all normal and a routine echocardiogram on the neonatal unit shortly after birth was normal. She attends the community clinic for the first time at 4 weeks of age. She is now thriving and feeding well. Her parents have many questions about what is going to happen in the future. In particular, they have been reading that she is at increase risk of certain diseases because she has Down syndrome.

Out of the following, which is a higher risk to Mary, compared with the general population?

Duodenal atresia
Ischaemic heart disease
Pyloric stensosis
Congenital heart disease
Leukaemia
A

Leukaemia.

Children with Down syndrome are also at an increased risk of congenital heart disease and duodenal atresia. However Mary had a normal echocardiogram and duodenal atresia would have already presented with bile-stained vomiting within the first few days of life. Children with Down syndrome are not at an increase risk of ischaemic heart disease or pyloric stenosis.

How well did you know this?
1
Not at all
2
3
4
5
Perfectly
30
Q

Louise has Turner’s syndrome. What is her karyotype?

46, XO
46, XX
45, XO
45, XY
47, XXY
A

45, XO.

There are 22 pairs of autosomes, and only one sex chromosome, an X. This makes the total number of chromosomes 45.

How well did you know this?
1
Not at all
2
3
4
5
Perfectly
31
Q

Louise is a 14-year-old girl. She is attending the endocrinology clinic, as she is the shortest girl in her class and wants to know if there is treatment to make her taller. She is otherwise well and has no other medical complaints. She as always been the shortest girl in her class. Her clinical appearance is shown in the picture (round facies with short, webbed neck). Her height plots well below the 0.4th centile but her weight is on the 9th centile. Her karyotype is 46, XX.

Select the most likely cause of her short stature.

Normal variant
Turner syndrome
Noonan syndrome
Cushing syndrome
Down syndrome
A

Noonan syndrome.

She has a round facies with short, webbed neck and short stature since infancy, characteristic of Noonan syndrome. There is some overlap with the phenotype of Turner syndrome, but her karyotype is normal. Cushing syndrome – there is growth failure.

How well did you know this?
1
Not at all
2
3
4
5
Perfectly
32
Q

A mother has just found out she is pregnant and asks for advice about how to look after her health and nutrition during pregnancy and to provide the best possible care for her newborn baby. She smokes 15 cigarettes a day. You recommend she gives up smoking.

Select the problem that has been linked to smoking.

Increased risk of shoulder dystocia
Increased risk of baby that is small for gestational age
Increased risk of Vitamin A deficiency
Increased risk of neural tube defects
Increased risk of dysmorphic syndromes
A

Increased risk of baby that is small for gestational age

Smoking reduces birthweight and is associated with increased risk of stillbirths and miscarriages.

How well did you know this?
1
Not at all
2
3
4
5
Perfectly
33
Q

A mother has her routine 20-week antenatal scan. The sonographer finds the fetal abdominal and head circumference measurements are normal, but is concerned that there is an abnormally small amount of amniotic fluid (oligohydramnios).

Select the most likely cause for this.

Maternal diabetes
Duodenal atresia
Gastroschisis
Poorly functioning kidneys
Severe intrauterine growth restriction
A

Poorly functioning kidneys

Oligohydramnios, reduced or lack of amniotic fluid, may be associated with decrease fetal urine output because of abnormal kidneys.

Oligohydramnios may be associated with severe intrauterine growth restriction, but the sonographer has identified a normal head and abdominal circumference.
Duodenal atresia, Maternal diabetes may be associated with polyhydramnios (excess amniotic fluid).
Gastroschisis should not affect the volume of amniotic fluid.

How well did you know this?
1
Not at all
2
3
4
5
Perfectly
34
Q

A midwife is concerned that a mother who is at 32 weeks’ gestation has a symphysis-fundal height smaller than expected. An ultrasound confirms intrauterine growth restriction.

Select the feature that would concern the sonographer that there may be fetal compromise.

Reverse end diastolic flow in the umbilical artery
Active fetal movements
Accelerations of metal heart rate
Normal amniotic fluid volume
Fetal breathing movements seen
A

Reverse end diastolic flow in the umbilical artery

Absent or reversed flow velocity during diastole carries an increased risk of morbidity from hypoxic damage to the gut or brain, or of intrauterine death.

Fetal breathing movements, accelerations on the cardiotocography (CTG) trace, active fetal movements and a normal amniotic fluid volume are a normal sign.

How well did you know this?
1
Not at all
2
3
4
5
Perfectly
35
Q

A mother has just found out she is pregnant with twins. Her antenatal scan reveals dichorionic, diamniotic twins.

Select the risk which is increased because of her twin pregnancy.

Twin to twin transfusion
Macrosomia
Gestational diabetes
Post-term gestation
Congenital abnormalities
A

Congenital abnormalities

There is a higher rate of congenital abnormalities in twin pregnancies. These occur twice as frequently as in a singleton (but the risk is increased four-fold in monochorionic twins).

Twin-twin transfusion syndrome is found in monochorionic twins (with a shared placenta). She has dichorionic diamniotic twins and therefore they are not at risk of this as they will have two separate placentas.

Multiple births are at risk of intrauterine growth restriction (IUGR) rather than macrosomia.

Multiple births are at risk of pre-term rather than post-term births. The median gestation for twins is 37 weeks.

How well did you know this?
1
Not at all
2
3
4
5
Perfectly
36
Q

You perform a routine newborn examination on a baby who is 20 hours old.

Which one of the following is true?

An absent red reflex is an indication for routine ophthalmological referral as an outpatient
An undescended testis requires urgent surgical repair
A heart murmur is physiological on the first day of life
The palate should be assessed by palpation only
Breast enlargement in newborn babies of either sex does not require investigation

A

Breast enlargement in newborn babies of either sex does not require investigation

Breast enlargement can occur in either sex and a small amount of milk may be discharged. This resolves spontaneously and does not require investigation.

An absent red reflex could be caused by cataracts or a retinoblastoma and needs an urgent ophthalmological referral.

Urgent surgical opinion for an undescended testis is not indicated as most descend spontaneously in the first year of life and surgical repair is therefore delayed. The testis should be rechecked at several months of age.

Although many heart murmurs heard at the first day of life are innocent and will disappear, some are from congenital heart disease.

Examination to exclude a cleft palate should be by palpation and visualisation of the palate.

How well did you know this?
1
Not at all
2
3
4
5
Perfectly
37
Q

An Afro-Caribbean mother, who is not known to have diabetes mellitus, is found to have glycosuria at her midwife appointment at 32 weeks’ gestation. Her glucose tolerance test and fasting glucose is abnormal. She is given dietary advice to control her blood sugars.

What is her newborn baby at increased risk of?

Small for gestational age
Respiratory distress syndrome
Hyperglycaemia at birth
Anaemia
Type 1 DM
A

Respiratory distress syndrome

This mother has gestational diabetes which places the infant at increased risk of respiratory distress syndrome as lung maturation is delayed.

Transient hypoglycaemia is common during the first day of life due to fetal hyperinsulinism, which is a response to the high levels of maternal glucose crossing the placenta.
These newborn infants are at increased risk of polycythaemia (venous haematocrit >0.65) rather than anaemia.
There is no increased risk of type 1 diabetes mellitus in babies born to mothers with gestational diabetes.

How well did you know this?
1
Not at all
2
3
4
5
Perfectly
38
Q

In the fetus, the lungs are filled with fluid and oxygen is supplied by the placenta. To enable the baby to adapt to extrauterine life, the fetus undergoes physiological changes during labour and in the first few minutes of life.

Which of the following is not true?

Oxygenated blood flow through the ductus arteriosus causes closure
Lung liquid is drained during descent through the birth canal
Pulmonary vascular resistance increases at birth
Foramen ovale closes due to increased left atrial pressure
Tactile stimuli at birth help initiate breathing

A

Pulmonary vascular resistance increases at birth

This is not true. Pulmonary vascular resistance falls on inflation of the lungs. There is therefore increased blood flow through the lungs, which leads to increased left atrial filling. The increased pressure in the left atrium causes the foramen ovale to close.

How well did you know this?
1
Not at all
2
3
4
5
Perfectly
39
Q

Jonathon, a newborn Caucasian baby, is noted to have hepatosplenomegaly and a petechial rash. His red eye reflex is normal and there is no heart murmur. He fails his newborn screening hearing test. His mother is from the UK and her antenatal screening bloods were all normal.

What is the most likely congenital infection that has caused these symptoms?

CMV
Rubella
Varicella zoster
Toxoplasmosis
Syphilis
A

CMV

This is the most common congenital infection in the UK. Most babies born to mothers with CMV infection during pregnancy are normal at birth, but 5% have clinical features such as hepatosplenomegaly and petechiae. Most of these babies will go on to have neurodevelopmental disabilities.

Congenital rubella can cause similar signs but is often associated with cataracts and congenital heart disease as well as deafness; congenital rubella is extremely rare in the UK since the MMR vaccine was introduced, and all mothers are screened for rubella antibodies.

Babies whose mothers develop chickenpox in the first 20 weeks of pregnancy are at risk of severe scarring of the skin and possibly ocular and neurological damage.

Babies with toxoplasmosis can have some of the features described, with the addition of intracranial calcification, hydrocephalus and retinopathy, but this infection is extremely rare in the UK.

This is unlikely because mothers in the UK are screened for syphilis infection, and we know that this mother’s screening bloods were normal.

How well did you know this?
1
Not at all
2
3
4
5
Perfectly
40
Q

A newborn Caucasian girl is having her routine neonatal examination at 24 hours of age. She was born at term by normal vaginal delivery. Examination is normal including her tongue, which is pink, but her hands and feet are still blue.

What investigation do you need to do?

None
Check O2 saturation
Four limb blood pressures
Echocardiogram
Chest X-ray
A

None

Hands and feet being blue (peripheral cyanosis) is very common in the first days of life. If the rest of the examination is normal, especially that the tongue is pink (i.e. no central cyanosis), no investigations are indicated and the parents can be reassured.

Four limb blood pressures are useful for helping to detect coarctation of the aorta. Peripheral cyanosis is not a sign of this. In addition, examination of the baby (which must include auscultation of the heart and palpation of the femoral pulses) is normal.

This baby is not centrally cyanosed and has a normal examination. Therefore there is no sign of cardiac disease, and the baby does not need an echocardiogram.

A chest X-ray would detect cardiac or respiratory disease, and this baby has a normal examination with no central cyanosis. A chest X-ray is therefore not indicated.

Oxygen saturations are likely to be normal in this child who does not have central cyanosis.

How well did you know this?
1
Not at all
2
3
4
5
Perfectly
41
Q

You are asked to review a baby by a midwife who has recently started doing routine examinations of the newborn. The baby was an unexpected breech presentation but had no neonatal problems. When she examined the hips she felt the left hip was ‘clicky’. When you examine the hips, you find that both hips abduct fully and the head of femur is stable and was not dislocated or dislocatable. There is no family history of developmental dysplasia of the hip.

What investigation and follow up does this child need?

None
Hip US in 6 weeks as an outpatient
Hip US immediately
Hip US immediately and referral to orthopaedic surgeon
Hip US immediately and referral to physio

A

Babies with breech presentation or where there is a positive family history are at increased risk of developmental dysplasia of the hip (DDH). In many hospitals these infants are recommended to have a hip ultrasound at about 6 weeks of age.

Babies with a breech presentation are at an increased risk of developmental dysplasia of the hip and this should be looked for.

Although this baby is at risk of developmental dysplasia of the hip because of the breech presentation, the hips are stable on examination and investigations do not need to be completed urgently. ‘Clicky’ hips are a common and benign finding on examination.

How well did you know this?
1
Not at all
2
3
4
5
Perfectly
42
Q

A mother is known to have pre-eclampsia and her fetus has shown signs of intrauterine growth restriction on antenatal scans. He is delivered at 37 weeks and weighs 2.2kg. He is admitted to the Special Care Baby Unit because of his size. He appears well and has had a breast-feed.

What is he most at risk of?

Group B strep infection
Hypoglycaemia
Hypercalcaemia
Anaemia
Congenital cardiac abnormality
A

Hypoglycaemia

The baby has intrauterine growth restriction. These babies are liable to hypoglycaemia from poor fat and glycogen stores.

This baby has intrauterine growth restriction. However, this does not increase the baby’s risk of Group B streptococcus infection compared to babies of normal weight.
These babies are at risk of hypocalcaemia rather than hypercalcaemia.
These babies are at risk of polycythaemia rather than anaemia.
Congenital cardiac anomalies are not caused by intrauterine growth restriction.

How well did you know this?
1
Not at all
2
3
4
5
Perfectly
43
Q

Severe hearing impairment can be detected by screening newborns using evoked otoacoustic emission.

Which newborn infants should be tested?

All newborns
Only newborns with a FHx of deafness
Only newborns whose parents are worried about their hearing
No newborns, as hearing screening can be performed more reliably at 8 months of age
Only newborns who are found to have other abnormalities

A

All newborns

In the UK, all newborns are now routinely screened at birth or within the first few days of life.

How well did you know this?
1
Not at all
2
3
4
5
Perfectly
44
Q

All newborn babies have a heel-prick blood sample taken at 5–9 days of age for biochemical screening (Guthrie test).

Which of the following is not tested for?

Cystic fibrosis
Phenylketonuria
Hypothyroidism
Sickle cell
Duchenne muscular dystrophy
A

Duchenne muscular dystrophy

How well did you know this?
1
Not at all
2
3
4
5
Perfectly
45
Q

Callum, a two-day-old Caucasian infant, weighs 3.6 kg at birth. He was born by vaginal delivery with Apgar scores of 7 at 1 minute and 10 at 5 minutes. On day 2, he is reported to be jittery, crying inconsolably and feeding poorly. He sneezes and yawns, and is thought to have some abnormal movements, possibly seizures. No dysmorphic features are present and he is not jaundiced.

What is the most likely explanation for Callum’s problems?

Congenital rubella syndrome
Fetal alcohol syndrome
Birth asphyxia
Kernicterus
Maternal opiate use
A

Maternal opiate use

The clinical features are consistent with this condition.

How well did you know this?
1
Not at all
2
3
4
5
Perfectly
46
Q

Mohammed is a 24-hour-old baby. His mother develops chicken pox (varicella) one day after his delivery.

What is the most appropriate advice you would give?

Reassure and discharge home to avoid spread of infection
Neonatal infection is unlikely due to transplacentally acquired antibodies
There is a significant risk of serious neonatal infection
Breast-feeding is contraindicated
The infant’s varicella antibody status should be checked

A

There is a significant risk of serious neonatal infection

If the mother develops chickenpox from five days before until 2 days after delivery, there will be insufficient time for protective antibodies to develop and be transferred to the infant. A quarter of such infants become infected with a significant mortality.

The infant will not have received protective antibodies against varicella from the mother, and is therefore at risk of morbidity and mortality if discharged home.

The mother is unlikely to have antibodies to varicella, because she herself has developed chickenpox. Transplacental transfer of antibodies will therefore not have happened.

Antibodies acquired via breast-milk are important for an infant’s immunity, although they will not protect against this episode of varicella.

It is unlikely that the mother will have any antibodies to varicella, as she herself has developed chickenpox. Therefore transplacental transfer of antibodies cannot have occurred, and you can be sure that the infant is not immune to varicella.

How well did you know this?
1
Not at all
2
3
4
5
Perfectly
47
Q

You are called as an emergency to delivery suite just as a term baby boy is being delivered. There had been an abnormal CTG (cardiotocography) prior to delivery, and a severe antepartum haemorrhage. At birth the baby is not breathing, has a heart rate less than 60 beats/minute and is white. Resuscitation is commenced and his Apgar scores are 2 at 1 minute, 4 at 5 minutes, and 4 at 10 minutes of age. He is transferred to the neonatal unit where he requires ventilatory support as he is not breathing adequately. His limbs are hypotonic, he has a seizure at 12 hours and then a further one at 18 hours of age.

He fulfils the definition of birth asphyxia because his Apgar score at 1 minute of age was less than 3
His Apgar score at 10 minutes of age is associated with an increased risk of long-term disability
He is too small to have an MRI scan to determine long-term prognosis
His clinical features are those of moderate hypoxic ischaemic encephalopathy
This baby is representative of the commonest cause of cerebral palsy

A

His Apgar score at 10 minutes of age is associated with an increased risk of long-term disability

Hypoxic ischemic encephalopathy is the name of the condition assigned to a hypoxic insult before, during or after delivery resulting in injury to the brain. It is caused by compromised placental or pulmonary gas exchange that results in cardio-respiratory depression. This leads to hypoxia, hypercarbia and decreased cardiac output.

If the Apgar score remains low at 10 minutes of age the risk of long-term disability or mortality is nearly 50%.

How well did you know this?
1
Not at all
2
3
4
5
Perfectly
48
Q

A baby girl is delivered by caesarean section at 32 weeks’ gestation because of maternal pre-eclampsia. Her birth weight is 1.9 kg. No resuscitation is required. At 2 hours of age she develops respiratory distress, with a respiratory rate of 70 breaths/minute, grunting respirations and indrawing of her rib cage. Respiratory support with CPAP (continuous positive airway pressure) and 45% oxygen is required. What is the most likely diagnosis?

Meconium aspiration
Pneumonia
Resp. distress syndrome
Transient tachypnoea of the newborn
Pneumothorax
A

Respiratory distress syndrome

Caused by a deficiency of surfactant, produced by type II pneumocytes, mainly in pre-term infants. It results in ventilation-perfusion mismatch. The chest X-ray characteristically shows a diffuse granular or ‘ground glass’ appearance of the lung fields, an air bronchogram outlining the larger airways, and the heart border is indistinct.

How well did you know this?
1
Not at all
2
3
4
5
Perfectly
49
Q

Robert is a full-term baby boy, born 10 hours ago. His mother is blood group O negative, and her membranes ruptured 2 days before delivery. He is breast-feeding well but the midwife noticed he looks markedly jaundiced. On examination the baby is clinically well but markedly jaundiced.

What investigation should be performed first?

Bilirubin level
Blood culture
Congenital infection screen
Direct antibody test
Blood group
A

Bilirubin level

Jaundice starting at less than 24 hours of age is most likely due to haemolysis and may rapidly rise to dangerously high levels. It needs urgent assessment and close monitoring. The most urgent investigation is to measure the bilirubin level, as this will determine the management required.

Direct antibody test is positive with Rhesus disease and ABO incompatibility.
Blood group will inform if ABO incompatibility is a potential cause.
Congenital infection can cause early jaundice but there are usually clinical features and the jaundice is mild.
Blood cultures are taken to exclude infection but do not influence immediate management.

How well did you know this?
1
Not at all
2
3
4
5
Perfectly
50
Q

A 3-week-old baby boy, at home with his mother, is noted to be jaundiced by the midwife. He is exclusively breast-fed and is growing well. When you examine him you note he is jaundiced but otherwise the examination is normal.

What is the most important investigation to do first?

Abdo USS
Unconjugated and conjugated bilirubin
No investigation needed
Unconjugated bilirubin alone
Congenital infection screen
A

Unconjugated and conjugated bilirubin

Jaundice in babies over 2 weeks of age must be investigated. Those who are breast-fed are most likely to have unconjugated hyperbilirubinaemia secondary to ‘breast-milk jaundice’. However, it is extremely important to rule out a conjugated cause, especially biliary atresia, which needs prompt diagnosis. The first investigation would be to measure the unconjugated and conjugated bilirubin.

How well did you know this?
1
Not at all
2
3
4
5
Perfectly
51
Q

You are asked to review a newborn baby who is only 24 hours old and has developed very swollen eyelids with a purulent discharge. The eyes have been swabbed and the gram stain comes back as gram negative bacteria.

What treatment is required?

Clean with cool boiled water
Reassure it will resolve spontaneously
Topical antibiotic ointment alone
IV antibiotics
Oral antibiotics
A

IV antibiotics

Purulent discharge and eyelid swelling in the first 48 hours of life must be taken seriously. The gram stain makes gonococcal infection likely and this should be treated promptly, e.g. with a third-generation cephalosporin intravenously; as permanent loss of vision can occur. The most common cause of severe neonatal purulent conjunctivitis is Chlamydia trachomatis.

How well did you know this?
1
Not at all
2
3
4
5
Perfectly
52
Q

Isabelle was born at term weighing 4kg. At 6 hours of age she was noted to be breathing fast and have a low temperature. She was born by normal vaginal delivery and the membranes had ruptured 24 hours previously. Mother had a low-grade fever and there was fetal tachycardia prior to delivery. Isabelle has not breast-fed since birth and has had one vomit. On examination, her temperature is 35.5°C, respiratory rate 90 breaths/minute. Capillary refill time is 3 seconds. She is lethargic. Her oxygen saturation is 89% in air.

Her chest X-ray shows consolidation at the right base. The microbiologist has phoned to say her blood cultures are growing an organism.

What is the most likely organism?

Listeria monocytogenes
Group B Streptococcus
Escherichia coli
Staphylococcus aureus
Herpes simplex virus unfection
A

Group B Strep

Isabelle has early-onset sepsis and this can be caused by infection in the urine, chest or cerebrospinal fluid (meningitis). The most common organism causing early-onset sepsis in the UK is Group B streptococcus.

Listeria monocytogenes, E. coli and Staphylococcus aureus may cause early-onset sepsis but are less common causes in the UK.

How well did you know this?
1
Not at all
2
3
4
5
Perfectly
53
Q

James was born at 39 weeks’ gestation by elective caesarean section because of pre-eclampsia. His birth weight was 3.7 kgs. His mother breast-fed him as soon as she had recovered from general anaesthetic. He fed well, but is vomiting after every feed. He is now 18 hours old and after the last 2 feeds he ‘vomited everything up’ and it was greenish in colour. On examination his temperature is 36.5°C, and he is alert and hungry.

His abdomen is not distended. He has not yet passed meconium.

What is the most likely diagnosis?

Pyloric stenosis
Duodenal atresia
Meconium ileus
Hirschprung disease
Neonatal sepsis
A

Duodenal atresia

This is the most likely cause of persistent bilious vomiting on the first day of life.

Pyloric stenosis: the vomit is not bile-stained as the obstruction is above the ampulla of Vater and presents at about 6 weeks of age.

Hirschprung disease: as this affects the rectum and sometimes the colon, this distal bowel obstruction results in marked abdominal distension.

Meconium ileus: this affects the lower ileum, so there would be abdominal obstruction.

Neonatal sepsis: can cause vomiting, sometimes slightly bile-stained, but there are no risk factors from labour and delivery, and no other features on clinical examination.

How well did you know this?
1
Not at all
2
3
4
5
Perfectly
54
Q

Aaron was born at term, birth weight 3.2 kg. He is now 2 weeks old, and his mother is concerned that his umbilicus looks abnormal. A photo is shown in the figure below.

Select the most likely diagnosis.

A

Umbilical granuloma- In umbilical granuloma there is a pink, pedunculated lesion of granulation tissue as shown here.

In umbilical hernia there is protrusion of the umbilicus.

In exomphalos the abdominal contents protrude through the umbilical ring, covered by the amniotic membrane and peritoneum.

In umbilical infection (omphalitis) there is redness of the skin surrounding the umbilicus, and may be a purulent discharge on the umbilicus.

In gastroschisis the bowel protrudes through a defect in the anterior abdominal wall adjacent to the umbilicus.

How well did you know this?
1
Not at all
2
3
4
5
Perfectly
55
Q

Janine, a 9-month-old Caucasian girl, is seen by her family doctor because of concern that she is not growing fast enough. She is only on the 5th centile for height and 2nd centile for weight.
What is the greatest influence on her growth rate at her age?

Genes
Nutrition
Growth hormone
Oestrogen
Testosterone
A

Nutrition

Along with good health, well-being and thyroid hormones, infantile growth (from birth to 12 months of age) is dependent on good nutrition.

How well did you know this?
1
Not at all
2
3
4
5
Perfectly
56
Q

John, a 3-year-old boy, is referred to paediatric outpatients because of concern about the development of pubic and axillary hair. His testes are 1.5 ml in size (pre-pubertal). His blood pressure is 100/75. Gonadotrophin levels are normal for a pre-pubertal boy (undetectable) but his bone age is 5 years.
What is the most likely cause of his early pubertal development?

Adrenal tumour
Brain tumour
Idiopathic precocious puberty
Testicular tumour
Prader-Willi syndrome
A

Adrenal tumour

Premature sexual development in boys is uncommon and usually has an organic (rather than constitutional or familial) cause. With his pre-pubertal testes, hypertension and normal gonadotrophin levels, the abnormality is likely to be in his adrenal glands.

How well did you know this?
1
Not at all
2
3
4
5
Perfectly
57
Q

Chelsea, a 7-year-old girl, is brought by her mother to the paediatric clinic. Her mother is concerned about Chelsea’s early development of puberty. Chelsea has started to develop breasts and, more recently, some pubic hair. On examination she has breast development stage BIII and pubic hair development PHII. She has not started her periods, but has had a growth spurt recently. An ultrasound of her pelvis shows multicystic ovaries and enlarging uterus.
What is the most likely cause of her early pubertal development?

Brain tumour
Ovarian tumour
Idiopathic precocious puberty
Turner syndrome
Congenital adrenal hyperplasia
A

Idiopathic precocious puberty

Precocious puberty in females is usually due to premature onset of normal puberty. The sequence of puberty in this child is normal and there is also been an associated growth spurt, which makes an idiopathic cause for the precocious puberty more likely. The pelvic ultrasound findings are consistent with premature onset of normal puberty.

Turner syndrome is associated with delayed rather than precocious puberty.

In congenital adrenal hyperplasia, the sequence of pubertal changes is abnormal, with isolated pubic hair and virilisation of genitalia.

How well did you know this?
1
Not at all
2
3
4
5
Perfectly
58
Q

Sophia, an 18-month-old girl, is brought to outpatients by her mother, who is very worried as she has developed breasts. She is otherwise well and has been growing normally. On examination she has breast development BIII but no pubic or axillary hair. Her bone age is 20 months.
What is the most likely cause of her early pubertal development?

Idiopathic precocious puberty
Premature thelarche
Premature pubarche
Brain tumour
Congenital adrenal hyperplasia
A

Premature thelarche

Sophia has breast development and no other signs of puberty. She has premature thelarche.

Sophia does not have precocious puberty because she does not have axillary or pubic hair, nor has she had a growth spurt.

Congenital adrenal hyperplasia would cause premature pubarche rather than isolated breast development.

She has not developed pubic or axillary hair.

A brain tumour is unlikely as she has isolated breast development.

How well did you know this?
1
Not at all
2
3
4
5
Perfectly
59
Q

Hypoglycaemia
George, a 2-week-old Caucasian baby boy, presents to the Accident and Emergency department with vomiting, diarrhoea and poor feeding.

Investigations show:
• Sodium 112 mmol/L (normal range 133-145 mmol/L)
• Potassium 6.8 mmol/L (normal range 3.5–6.0 mmol/L)
• Urea 7.8 mmol/L (normal range 2.5–8.0 mmol/L)
• Creatinine 30 μmol/L (normal range 20–65 mmol/L)
• Blood glucose 1.7 mmol/L (normal range >2.4 mmol/L)
• Infection screen—negative

What is the most likely cause?

Congenital adrenal hyperplasia
Acute renal failure
Hypothyroidism
Cushing syndrome
Gastroenteritis
A

Congenital adrenal hyperplasia

The combination of hyponatraemia, hyperkalaemia and hypoglycaemia are suggestive of glucocorticoid deficiency, as seen in congenital adrenal hyperplasia.

Gastroenteritis can lead to hyponatraemic dehydration but the urea would be raised and the potassium normal.

An excess of glucocorticoids, as seen in Cushing syndrome, causes an intolerance to carbohydrates (hyperglycaemia), rather than the hypoglycaemia seen in this child.

Hyponatraemia may be a feature of acute renal failure, but this child has a normal plasma urea and creatinine.

How well did you know this?
1
Not at all
2
3
4
5
Perfectly
60
Q

A newborn baby has recently been delivered. The midwife requests an urgent paediatric review of the baby because she cannot tell if the baby is male or female. The parents are asking what sex their baby is.
What should you tell them?

You are unable to tell right now but will be able to assign a sex as soon as you get the baby’s chromosomes back
You are unable to tell right now and a detailed assessment of the baby including scans and blood tests will be needed before you can tell them
You feel it is likely to be a girl so tell them it is female
You feel it looks like boy so tell them he is male
You are unable to tell if the baby is male or female and tell the parents it is likely to be a mixture of both sexes i.e. ovotesticular disorder of sex development (DSD – or hermaphroditism).

A

You are unable to tell right now and a detailed assessment of the baby including scans and blood tests will be needed before you can tell them

A detailed assessment by medical, surgical and psychological specialists needs to be performed followed by a full discussion with the parents before the infant is assigned a sex

If there is abnormal sexual differentiation at birth, do not guess the infant’s gender. A detailed assessment needs to be performed.

How well did you know this?
1
Not at all
2
3
4
5
Perfectly
61
Q

A Caucasian infant is born with a disorder of sexual differentiation. Congenital adrenal hyperplasia is suspected.
What blood result would confirm the diagnosis?

A markedly raised plasma 17α-hydroxyprogesterone
A markedly lowered plasma 17 α-hydroxyprogesterone
A low testosterone
A markedly raised cortisol level
A raised blood sugar

A

A markedly raised plasma 17α-hydroxyprogesterone

Most children with congenital adrenal hyperplasia have 21-hydroxylase deficiency. The diagnosis is made by finding markedly raised levels of the metabolic precursor 17 α-hydroxy-progesterone.

In congenital adrenal hyperplasia the cortisol level would be low and the testosterone would be raised..

How well did you know this?
1
Not at all
2
3
4
5
Perfectly
62
Q

Babatunde, a 13-month-old black African boy, presents to the paediatric clinic with progressive bowing of his legs. He is still entirely breast-fed. He is miserable, and his wrists are noted to be swollen. An X-ray is taken of his wrists and is shown in the figure below. What is the most likely diagnosis?

Vit A deficiency
Vit B1 deficiency
Vit D deficiency
Vit E deficiency
Vit K deficiency
A

Vit D deficiency

He has Vitamin D deficiency resulting in rickets. This has caused bowing of his legs; also the ends of the radius and ulna are expanded and rarified, and cup-shaped. At 13 months of age, breast-milk alone does not provide adequate intake of vitamin D.

How well did you know this?
1
Not at all
2
3
4
5
Perfectly
63
Q

Sarah, a 9-year-old Caucasian girl, is referred by the school nurse to the paediatric clinic because of her weight. She weighs 43 kg (98th centile) and is 141 cm tall (91st centile). She has followed her height centiles for the last 9 months, but her weight centile has increased. Her BMI is on the 97th centile.
Which of the following statements is true?

Sarah is obese
Sarah’s main problem is that she has a low metabolic rate
A BMI remaining at this level places Sarah at an increased risk of an abnormal lipid profile and raised blood pressure in adult life
Sarah’s adreno–cortical axis should be checked to exclude Cushing syndrome
A crash diet is the treatment of choice

A

Sarah is overweight (BMI >91st centile) rather than obese (BMI >98th centile). It places her at increased risk of an abnormal lipid profile and raised blood pressure in adult life.

For clinical use, obese children are those with a BMI above the 98th centile of the UK 1990 reference chart for age and sex. Sarah is overweight (BMI >91st centile).

Cushing syndrome is rare and associated with a failure of linear growth.

How well did you know this?
1
Not at all
2
3
4
5
Perfectly
64
Q

Regarding the Barker hypothesis relating birthweight to diseases in adult life, which of the following newborns have the highest risk of cardiovascular disease in later life?

Birthweight

A

Birthweight

How well did you know this?
1
Not at all
2
3
4
5
Perfectly
65
Q

Which of the following is not correct regarding the advantages of breast-feeding over formula-feeding?

Higher interferon level in breast-milk
Decreased risk of gastrointestinal infections associated with breast-feeding
Closer mother-child relationship with breast-feeding
Higher vitamin K concentration in breast-milk
Lengthened time-period between children

A

Higher vitamin K concentration in breast-milk
The vitamin K concentration in breast-milk is lower. There is insufficient vitamin K in breast-milk to reliably prevent haemorrhagic disease of the newborn. This risk is minimised by recommending prophylactic vitamin K to all newborns.

Breast-feeding leads to an increased time period between children because of its contraceptive effect. However, it is not a fully effective form of contraception, and counselled regarding contraceptive methods should be given.

Breast-feeding enhances mother-child relationship.

Interferon is present in breast-milk and has an antiviral property.

Breast-feeding is life-saving in developing countries due to the association with fewer gastrointestinal infections.

How well did you know this?
1
Not at all
2
3
4
5
Perfectly
66
Q

Anil is a 2½-year-old boy who lives in India and attends the local health clinic for a routine check. Both his parents are subsistence farmers. He is asymptomatic. On examination, he is very thin, but his hair and skin appear normal, and there is no oedema or other clinical abnormalities. His height is on the 5th centile but his weight is well below the 4th centile.
What is the most likely diagnosis?

Sever gastro-oesophageal reflux
Marasmus
Kwashiorkor
Rickets
Normal child
A

Marasmus

In marasmus, severe protein-energy malnutrition, the weight for height more than –3 standard deviations below the median, corresponding to

How well did you know this?
1
Not at all
2
3
4
5
Perfectly
67
Q

Harry is a 13-year-old Caucasian boy who attends the paediatric clinic because of obesity. His height is on the 98th centile, and his weight is above the 99.6th centile.
Which of the following is NOT associated with obesity in later life?

Type 1 Diabetes
Slipped upper femoral epiphysis
Hypertension
Asthma
Low self esteem
A

Type 1 Diabetes

Obese children are more likely to have non-insulin-dependent diabetes mellitus (type 2 diabetes). Type 1 diabetes is an autoimmune disorder and is not related to a child’s weight.

How well did you know this?
1
Not at all
2
3
4
5
Perfectly
68
Q

Benjamin is a 14-month-old boy, who is seen in the paediatric emergency department. He has been vomiting and has had diarrhoea for 3 days. Today, he has eaten much less food than usual and has vomited after each feed. His stool is watery and foul-smelling but has no blood in it. He has not been out of the UK since he was born. Examination shows that he is slightly dehydrated, but is otherwise normal.

What is the most likely organism that has caused his symptoms?

Rotavirus
Escherichia Coli
Shigella
Campylobacter
Giardia lamblia
A

Rotavirus

Rotavirus is a common cause of foul-smelling, watery diarrhoea. E. coli, Shigella and Campylobacter also cause explosive watery diarrhoea but can be associated with blood in the stools.

Giardia lamblia is a parasitic infection and is usually acquired whilst travelling abroad, although there are rare cases in the UK.

How well did you know this?
1
Not at all
2
3
4
5
Perfectly
69
Q

Emma is a 12-month-old Irish girl who is seen at the outpatient department. She presents with loose stools two to three times each day, with no blood in them. She is generally difficult, and it has become a battle to get her to feed. Examination of her abdomen, although limited due to distress, is unremarkable apart from being slightly distended. You suspect that her buttocks may be somewhat wasted. She is on no medications. Her weight was on the 9th centile and is now below the 0.4th centile

What is the most likely cause for the weight loss?

Lactose intolerance
Coeliac disease
Ulcerative colitis
Toddler diarrhoea
Hirschsprung disease
A

Coeliac disease

Coeliac disease (gluten-sensitive enteropathy) usually presents at age 8–24 months with abnormal stools, failure to thrive (growth faltering), abdominal distension, wasting of the muscles of the buttocks (a difficult clinical sign) and irritability. Can also present with short stature or anaemia. It is increasingly detected on screening high-risk groups, and at older ages.

70
Q

Rodney, a boisterous 2-year-old, has had diarrhoea for the last 3 months. He produces up to four stools a day which are loose, brown in colour and usually contain undigested carrots and other food. The rest of the family are well. He has never been abroad. Examination is normal and his personal child health record shows that he is growing along the 50th centile for weight and 75th centile for height.

What is the most likely diagnosis?

Coeliac disease
IBD
Toddler diarrhoea
Lactose intolerance
Cow's milk protein intolerance
A

In toddler diarrhoea there are loose stools with undigested food present. The children grow well and have plenty of energy.

71
Q

Ellie is a 4-year-old girl who has been complaining of pain in her tummy for a month. It is worse when she goes to the toilet; her stools are firm and she opens her bowels only every 2 – 3 days. She has not had any vomiting. For the last 2 weeks her stools have become loose. On examination she has a mass in the left iliac fossa.

What is the most likely diagnosis?

Wilms tumour
Constipation
Gastroenteritis
Appendix mass
Inguinal hernia
A

Constipation

The loose stool is overflow from her constipation. Children may present with loose stools when they actually have constipation.

Wilms tumour can present as an abdominal mass but would not be confined to the left iliac fossa, and is rare.

Gastroenteritis is unlikely as there is no vomiting and the problem has been going on for a month.

An inguinal hernia is not associated with loose stools. It may be associated with pain in the groin or abdomen if it is strangulated, which is not the case in this child. It is also much less common in girls than boys.

72
Q

Aiysha is a 2-month-old baby who is seen in the paediatric outpatient department. She was born at term, birthweight 3.5 kg, and is breast-fed. Her mother is concerned as she has vomited some of the milk after most feeds since birth, and this is messy and requires her outfits to be repeatedly changed and washed and the flat where they live smells of vomit. On several occasions the vomit has been blood-stained. She cries when she vomits. She is continuing to grow along the 50th centile.

What is the most likely diagnosis?

Pyloric stenosis
Helicobacter pylori infection
Infant colic
Gastro-oesophageal reflux
Recurrent UTIs
A

Gastro-oesophageal reflux

Caused by the involuntary passage of gastric contents into the lower oesophagus. These infants can vomit several times per day but still continue to gain weight appropriately. It is managed by positioning in a more upright position and by thickening feeds.

73
Q

Claire, a 7-year-old girl, has had abdominal pain for the last 6 months. On several occasions, it has been sufficiently severe for her be sent home early from school. The pain happens once or twice a week in the afternoon or early evening. It is periumbilical in nature. It does not wake her at night. She has not had vomiting or diarrhoea. She is growing well. Her examination is normal. Her urine is clear on dipstick testing.

What is the most likely cause for her pain?

Hepatitis
Functional abdominal pain
Peptic ulceration
IBS
Gastritis
A

Functional abdominal pain is classically periumbilical and is not associated with any other symptoms. The pain has been going on for 6 months and Claire is well in herself, which suggests that this is not pathological. The pain is very real but needs to be explored, as in some children it is a manifestation of stress.

74
Q

Ellie is a 12-year-old girl who has had loose stools, abdominal discomfort that improves when she opens her bowels, and feeling of having a bloated tummy for the last few months. A stool sample was sent for culture and this was negative. There has been no blood in her stool. She is otherwise well. Examination, including of the abdomen, is normal.

What is the most likely diagnosis?

IBS
Abdominal migraine
Appendicitis
Crohn's disease
Coeliac disease
A

IBS

75
Q

Ben, aged 9 months, has had a 3-day history of diarrhoea and vomiting. On examination he is found to be quiet but alert, is tachypnoeic, has a tachycardia but normal pulses, dry mouth, no mottling of the skin but reduced skin turgor and a sunken fontanelle. Capillary refill time is 2 seconds. His blood pressure is normal for his age. He continues to vomit even with oral rehydration solution given via a naso-gastric tube. Ben’s plasma sodium is found to be 156 mmol/L (normal range 135–145 mmol/L). He needs fluid as he is dehydrated. How quickly should this fluid be replaced?

Immediate bolus of IV fluid as he is so dehydrated
Rehydration should be rapid (48 hours) as the expansion of the brain cells should be corrected slowly
Rehydration should be slow (>48 hours) as the contraction of brain cells should be corrected slowly

A

Rehydration should be slow (>48 hours) as the contraction of brain cells should be corrected slowly

Ben has hypernatraemic dehydration so rehydration must be slow, i.e. over 48 hours to avoid cerebral oedema. In hypernatraemic dehydration the brain cells are contracted as water has moved to the extracellular compartment, as this has a higher osmolality (see figure below). Initially, fluids can be given nasogastrically over 6 hours, but if not tolerated, intravenous therapy should be started.

76
Q

Matthew is a 3-day-old term baby who has not passed meconium since birth. On examination his abdomen is distended but the remainder of the examination is normal. A rectal examination by the paediatric surgeons results in explosive loss of stool.

What is the most likely diagnosis?

CF
Constipation
Hirschsprung disease
Duodenal atresia
Rectal atresia
A

Hirschsprung disease

Caused by the absence of ganglion cells from the myenteric and submucosal plexuses of part of the large bowel which results in a narrow, contracted segment. The abnormal bowel extends from the rectum for a variable distance proximally, ending in a normally innervated, dilated colon.

77
Q

Henry is 4 years old and has a 3-day history of fever. He presents to the Accident and Emergency department with a headache. A lumbar puncture is performed. You receive the following result from the laboratory:

  • CSF microscopy: 1500 neutrophils
  • CSF protein: 0.6 g/L
  • CSF glucose: 2.1 mmol/L
  • Blood glucose: 7.2 mmol/L
The normal CSF values in a child are:
• Microscopy WBC 0–5 mm3
• RBC 0 mm3
• Protein 0.15–0.4 g/L
• Glucose >/= 50% blood glucose

What is the most likely diagnosis at this point?

Normal LP result
Bacterial meningitis
Blood-stained tap
Tuberculosis meningitis
Viral meningitis
A

Bacterial meningitis

There is a markedly raised white cell count, all neutrophils. The CSF protein is raised, with a reduced CSF glucose).

The white blood cells are predominantly lymphocytes rather than neutrophils in CSF in viral meningitis.

TB meningitis presents with lymphocytes rather than neutrophils in CSF, the CSF protein is markedly raised and the glucose very low.

78
Q

Graham is 5 years old and has had an intermittent fever for 4 weeks. He presents to the Accident and Emergency department with a headache and neck stiffness. A CT is performed which is normal. Therefore a lumbar puncture is performed. You receive the following result from the laboratory:

  • CSF microscopy: 95 lymphocytes, 10 neutrophils
  • CSF protein: 2.2 g/L
  • CSF glucose: 1.3 mmol/L
  • Blood glucose: 6.3 mmol/L
The normal CSF values in a child are:
• Microscopy WBC 0–5 mm3
• RBC 0 mm3
• Protein 0.15–0.4 g/L
• Glucose > 50% of blood glucose

What is the most likely diagnosis?

Ascending polyneuritis (Guillain-Barre syndrome)
Bacterial meningitis
Blood-stained tap
Tuberculosis meningitis
Viral meningitis
A

Tuberculosis meningitis

The white cell count shows predominantly lymphocytes. This could also be seen in viral meningitis and early stage of bacterial meningitis, but the very markedly raised protein and very low glucose in the CSF together with the clinical history are suggestive of TB.

In ascending polyneuritis (Guillain–Barre syndrome), the protein level would also be high but the white cells would not be raised and the glucose would not be low.

79
Q

Ahmed, a 1-year-old boy, is brought to the paediatric assessment room. He has had a temperature for the last week. His mother has taken him to the general practitioner on 2 occasions and he has completed a course of amoxicillin. His eyes are injected and his throat is red. He has marked cervical lymphadenopathy. His heart sounds are normal and his chest is clear. Palpation of his abdomen is unremarkable. The skin on his fingers has started to peel. Everyone else in the family is well.

What is the most likely diagnosis?

Staphylococcal scalded skin syndrome
Tuberculosis
Kawasaki disease
Streptococcal infection
Infectious mononucleosis
A

Kawasaki disease

He fulfils the major diagnostic criteria, of fever for more than 5 days, with cervical lymphadenopathy, injected pharynx, conjunctivitis and peeling fingers. In staphylococcal scalded skin syndrome, the skin peeling would be more widespread.

80
Q

Prince, a 4-month-old black African boy, who has recently moved to the UK from Swaziland with his mother, is seen urgently in the paediatric assessment unit. He is feeding poorly and has become very breathless. He has had loose stools for the last 4 weeks and has not put on any weight since then. He has not received any medical treatment. On examination he appears pale and has marked intercostal recession. His oxygen saturation in air is 82%.

Respiratory syncital virus
Parainfluenza virus
Rhinovirus
Pneumocystis carinii (jiroveci) pneumonia
Staphylococcal pneumonia
A

Pneumocystis carinii (jiroveci) pneumonia

This patient is from Swaziland, which markedly increases his risk of human immunodeficiency virus. All the other infections can cause respiratory failure but are less likely in this particular patient.
Prophylactic treatment with co-trimoxazole could have prevented this infection.

81
Q

You are in the baby clinic at the local GP practice. One of the mothers is questioning whether or not she should get her baby immunised, as she wonders if it is really necessary. He does not go to nursery and has no siblings so is not at risk of infection.

What advice would you give?

Immunisation is important, as a high proportion of children need to be immunised to remove the infections from the community
As most other children are immunised, it is not crucial for her child to be immunised as he is unlikely to be exposed to any of the infections
Immunisation is important to provide immunity for her child from serious infections, and to remove some of these infections from the community.
Vaccines are associated with side-effects, so you can understand her reasoning
It is her choice, and you do not feel you should give an opinion.

A

Immunisation is important to provide immunity for her child from serious infections, and to remove some of these infections from the community.

The national immunisation programme provides protection against a range of serious infections. Some are for protection from pathogens prevalent in the community, e.g. diphtheria and tetanus; others rely on both protection to the individual and herd immunity, e.g. whooping cough and measles.

82
Q

Which disease has been eradicated worldwide through immunisation?

Smallpox
Pertussis
Measles
Mumps
Tetanus
A

Smallpox

83
Q

You are in a general practice. A 4-week-old boy presents with a fever. He was in contact with a child with chickenpox 2 days before. He is breast-fed, and his mother is concerned he may not be getting as much milk as usual although there has not been any change in his feeding pattern. He is having the same number of wet nappies as before he became unwell. Examination shows a temperature of 37.9°C, and he watches you and is moving his arms and legs but you cannot elicit a smile when you smile at him. His respiratory rate is 40 breaths/minute, his pulse 139 beats/minute and capillary refill time 2 seconds. There are no abnormalities on examination.

You are concerned he may have a serious bacterial illness. Which of the following is the most important risk factor for him in having a serious illness?

His age
Temp 37.9 C
Not smiling at you
Contact with chickenpox
Capillary refill time
A

His age

84
Q

Jonathan, aged 6 years, is brought to the Accident and Emergency department after becoming unwell at a family party. He is unable to say more than a single word and he indicates that he is finding it hard to breathe. He is very anxious. He has swollen lips and a raised itchy rash that is spreading from his face down to his chest. He has never had an episode like this before. He has a history of asthma and is on a steroid inhaler which he uses via a spacer. His respiratory rate is 45 breaths/minute and inspiration is loud and laboured. His oxygen saturation in air is 90%.
What is the most likely diagnosis? Choose the single most appropriate answer.

Croup
Inhaled foreign body
Severe asthma
Meningococcal septicaemia
Anaphylaxis
A

Anaphylaxis

His stridor in association with angioedemia and urticaria are characteristic of anaphylaxis. The likely cause is a peanut or tree nut encountered at the party.

85
Q

Jonathan, aged 6 years, is brought to the Accident and Emergency department after becoming unwell at a family party. He is unable to say more than a single word and he indicates that he is finding it hard to breathe. He is very anxious. He has swollen lips and a raised itchy rash that is spreading from his face down to his chest. He has never had an episode like this before. He has a history of asthma and is on a steroid inhaler which he uses via a spacer. His respiratory rate is 45 breaths/minute and inspiration is loud and laboured. His oxygen saturation in air is 90%. Anaphylaxis is diagnosed.
What will be your immediate first step in his management?

Lie him flat
Separate him from his parents to keep him calm
Assess his airway and give him high-flow oxygen
Isolate him as he is likely to be infectious
Nebulised steroids

A

Assess his airway and give him high-flow oxygen

Jonathan requires oxygen as he has upper airways obstruction with low oxygen saturation.

86
Q

Jonathan, aged 6 years, is brought to the Accident and Emergency department after becoming unwell at a family party. He is unable to say more than a single word and he indicates that he is finding it hard to breathe. He is very anxious. He has swollen lips and a raised itchy rash that is spreading from his face down to his chest. He has never had an episode like this before. He has a history of asthma and is on a steroid inhaler which he uses via a spacer. His respiratory rate is 45 breaths/minute and inspiration is loud and laboured. His oxygen saturation in air is 90%. Anaphylaxis is diagnosed. You decide to assess his airway and give him high-flow oxygen.
What will be your next management step? Choose the single most suitable answer.

Third generation cephalosporin via an intravenous cannula
Nebulised salbutamol
Inhaled salbutamol via a large volume spacer
Intramuscular epinephrine (adrenaline)
Intravenous chlorpheniramine

A

Intramuscular epinephrine (adrenaline)

This is the immediate treatment he requires to reverse his acute laryngeal oedema, although nebulised or inhaled salbutamol and steroids may help his airways obstruction. Intravenous epinephrine (adrenaline) is too short-acting and has potentially serious side-effects.

87
Q

Jonathan, aged 6 years, is brought to the Accident and Emergency department after becoming unwell at a family party. He is unable to say more than a single word and he indicates that he is finding it hard to breathe. He is very anxious. He has swollen lips and a raised itchy rash that is spreading from his face down to his chest. He has never had an episode like this before. He has a history of asthma and is on a steroid inhaler which he uses via a spacer. His respiratory rate is 45 breaths/minute and inspiration is loud and laboured. His oxygen saturation in air is 90%. Anaphylaxis is diagnosed. You assess his airway and give him high-flow oxygen, before administering Intramuscular epinephrine (adrenaline).
What treatment should he be discharged home with?

Prednisolone
Antihistamine
Salbutamol inhaler
Adrenaline (epinephrine) auto-injector (e.g. Epipen)
All of the above
A

All of the above

A child who has had an anaphylactic reaction and has a history of asthma should have an adrenaline (epinephrine) auto-injector and all the treatment listed above, in case he has another anaphylactic reaction.
The underlying history of asthma predisposes him to bronchospasm.

88
Q

Eddy is an 8-month-old infant who presents to his general practitioner following a bout of diarrhoea and vomiting, associated with fever, for 3 days. This was managed at home with oral rehydration solution. The vomiting had settled but the diarrhoea has continued for a few weeks. The stool microscopy and culture were negative. He has no other medical problems and is not on any other medication. On examination he is not dehydrated. He has a soft, mildly distended abdomen.
What is the most likely cause?

Non-IgE-mediated cow’s milk protein allergy
IgE-mediated cow’s milk protein allergy
Non-allergic food hypersensitivity
Viral exanthema
Eczema
A

Non-allergic food hypersensitivity

This is likely to be temporary lactose intolerance (a non-allergic food hypersensitivity) following gastroenteritis. The stool is likely to have reducing substances in it. This should resolve over several weeks.

89
Q

Nagala, a 6-year-old, is taken by her mother to her general practitioner. She has been getting watery eyes and is often seen to vigorously rub her nose. On examination she has red eyes and is coryzal.

Allergic rhinitis and conjunctivitis
Eczema
Asthma
Croup
Inhalation of a foreign body
Drug allergy
Cow’s milk allergy
Lactose intolerance
Coeliac disease
A

Allergic rhinitis and conjunctivitis

This girl has an ’allergic salute’ from rubbing the itchy nose found in chronic allergy.

90
Q

Deepa, a 20-month-old Indian girl presents to the emergency department at 2 am. Her parents awoke to find her struggling to breathe. On examination she has stridor at rest and a barking cough. Her respiratory rate is 40 rpm and her oxygen saturation is 94%.

Allergic rhinitis and conjunctivitis
Eczema
Asthma
Croup
Inhalation of a foreign body
Drug allergy
Cow’s milk allergy
Lactose intolerance
Coeliac disease
A

Croup

91
Q

Fatima is a 6-month-old girl of Afro-Caribbean descent who lives in London. She was born at term by a vaginal delivery and has breast-fed well since then. She presents with a generalised, raised rash that she is scratching. Her mother is wanting to wean her and had introduced some yogurt into her diet earlier that day.

Allergic rhinitis and conjunctivitis
Eczema
Asthma
Croup
Inhalation of a foreign body
Drug allergy
Cow’s milk allergy
Lactose intolerance
Coeliac disease
A

Cow’s milk allergy

92
Q

Gina is 4 years old and presents to her general practioner with a cough. The cough is only present at night and has been there ’for months’. She had moderately severe eczema at 1 year of age and is allergic to cats and nuts. On examination she is well, with no focal chest signs.

Allergic rhinitis and conjunctivitis
Eczema
Asthma
Croup
Inhalation of a foreign body
Drug allergy
Cow’s milk allergy
Lactose intolerance
Coeliac disease
A

Asthma

93
Q

Priya, a 4-year-old Indian girl, presents to the emergency department with a cough. The cough developed suddenly after eating peanuts for the first time. She is allergic to penicillin and cats. On examination she has no audible stridor but is tachypnoeic. She continues to cough persistently.

Allergic rhinitis and conjunctivitis
Eczema
Asthma
Croup
Inhalation of a foreign body
Drug allergy
Cow’s milk allergy
Lactose intolerance
Coeliac disease
A

Inhalation of a foreign body

94
Q

Amber, a 9-month-old Caucasian girl, presents with a 4-day history of coughing spasms which are followed by vomiting. Whooping cough (pertussis) is suspected.

How would you best confirm the diagnosis?

By demonstrating a marked neutrophilia on a full blood count
By demonstrating a marked lymphocytosis on a full blood count
By immunofluorescence of a nasopharyngeal aspirate
By culturing a pernasal swab
By demonstrating patchy consolidation without collapse on a chest X-ray

A

By culturing a pernasal swab

Culturing a pernasal swab allows the pathogen (Bordetella pertussis) to be identified (though PCR is more sensitive).

95
Q

Tak, a 3-year-old Asian boy, presents to his family doctor. He has a ‘hacking’ cough that started several weeks ago, immediately after his birthday party and has failed to respond to two courses of antibiotics. He is otherwise well and has had no previous chest problems. On examination there is decreased air entry in the right lower zone with a normal percussion note. He is well grown.

Which is the most appropriate next course of action?

Organise for his pleural effusion to be tapped
A chest X-ray is indicated
A therapeutic trial of an inhaled bronchodilator is indicated
High-dose oral antibiotics should be prescribed
A sweat test should be obtained

A

A chest X-ray is indicated

The most likely diagnosis is the inhalation of a foreign body during his birthday party, e.g. a peanut. Tak also has focal chest signs and so needs a chest X-ray to be performed.

96
Q

Amir, a 3-year-old Asian boy, presents to his family doctor with a history of chronic nocturnal cough and intermittent wheeze.

Which of the following is the single most effective mode of bronchodilator delivery for Amir?

Dry powder inhaler
Metered dose inhaler alone
Metered dose inhaler with spacer
Tablets
Syrup
A

Metered dose inhaler with spacer

The best mode of delivery is direct to the lungs. Children under 5 years should be prescribed a metered dose inhaler (MDI) with spacer as they cannot coordinate a MDI alone.

97
Q

Sarah, a 10-year-old Caucasian girl, has frequent attacks of asthma and presents to Accident and Emergency with increasing difficulty in breathing over the last 12 hours. Initial observation shows that she is anxious, sitting upright, has a marked tracheal tug and is unable to complete a sentence.

Which of the following statements is true?

Sarah’s asthma attack is of moderate severity
If there is no audible wheeze, this is reassuring
Sarah’s condition is likely to improve if she is encouraged to lie flat
Sarah’s oxygen saturation should be measured
Sarah should be taken promptly to the X-ray department for a chest X-ray

A

Sarah’s oxygen saturation should be measured

Oxygen saturations should be measured to further assess the severity of the asthma attack, and to guide treatment.

98
Q

Zak, a 3-year-old Caucasian boy, is seen by his general practitioner because of recurrent wheezing associated with upper respiratory tract infections.

Which of the following features support the diagnosis of asthma?

Finger clubbing
Purulent sputum
Daytime cough unrelated to exercise
Static weight for the last 12 months
The presence of eczema
A

The presence of eczema

IgE-mediated asthma (atopic asthma) is more common in children with eczema.

99
Q

Boris, a 5-month-old boy from Poland, is admitted to hospital with breathing problems and poor feeding. On examination he has a respiratory rate of 50 breaths/minute with widespread crackles on auscultation of the chest, intercostal recession, and oxygen saturation of 92% in air. He was born at term with a birthweight of 3.6 kg (50th centile). His weight is now 5.2 kg (

A

Sweat test

Abnormal function of the sweat glands results in excessive concentrations of sodium and chloride in the sweat, and this is the basis of the essential diagnostic test for cystic fibrosis.

100
Q

Norah, an 18-month-old Caucasian girl, presents to her family doctor with coryza, cough and a mild fever for 3 days. She feeds poorly and is unsettled when febrile. Her respiratory rate and chest movements are normal, the oropharynx appears normal, as is auscultation of the chest.

What is the most likely diagnosis?

Upper respiratory tract infection
Frontal sinusitis
Tonsillitis
Bronchiolitis
Pneumonia (lower respiratory tract infection)
A

Upper respiratory tract infection

This child has an upper respiratory tract infection, most likely a common cold due to the nasal discharge and nasal blockage, which causes poor feeding.

101
Q

Jake is a 1-year-old boy from the UK who presents to the Accident and Emergency department with a 2-day history of fever and runny nose. He has been otherwise well. During the night he gradually developed a harsh cough in association with a loud noise on inspiration. On examination he has noisy inspiration accompanied by marked sternal recession, his capillary refill time is normal and he has a temperature of 38°C.

Which is the most likely diagnosis?

Acute epiglottitis
Anaphylaxis
Exacerbation of asthma
Laryngotracheobronchitis (croup)
Laryngeal foreign body
A

Laryngotracheobronchitis (croup)

Laryngotracheobronchitis (croup) is mucosal inflammation and increased secretions affecting the airway. Croup occurs from 6 months to 6 years of age. There is marked stridor.

102
Q

Muhammed is a 1-year-old boy old who has recently moved to the UK from Somalia. He presents to the Accident and Emergency department. He has been well today with the child-minder. This evening he suddenly developed a fever and marked difficulty breathing. He has a quiet cough and noisy inspiration. He has a temperature of 40°C and a capillary refill time of 3 seconds. He has severe intercostal and subcostal recessions.

Which is the most likely diagnosis?

Acute epiglottitis
Anaphylaxis
Exacerbation of asthma
Laryngotracheobronchitis (croup)
Laryngeal foreign body
A

Acute epiglottitis

Muhammed is very unwell with a fever, respiratory distress and a prolonged capillary refill time. In the UK and many other countries, the introduction of Hib (Haemophilus influenza type B) immunisation in infancy has meant that epiglottitis is now extremely rare.

103
Q

Alan, a 4-month-old boy, sees his general practitioner for an ear infection. On listening to his chest a heart murmur is heard.

Which one of the following features suggests that it requires further investigation?

A thrill
Sinus arrhythmia
Disappearance of murmur on lying flat
Systolic
Murmur maximal at the left sternal edge
A

A thrill

A thrill is a palpable murmur i.e. a loud murmur.
It always requires further investigation.

104
Q

Jo, a 3-month-old girl, presents with heart failure. A ventricular septal defect is identified on echocardiography.

Which of the following statements is correct?

The louder the murmur, the larger the defect
A soft second heart sound indicates pulmonary hypertension
A small left-to-right shunt is more likely than a large shunt to result in pulmonary hypertension
This child is at risk of Eisenmenger syndrome
Antibiotic prophylaxis should be started to prevent bacterial endocarditis

A

This child is at risk of Eisenmenger syndrome

Eisenmenger syndrome (irreversible damage to the pulmonary vascular system) usually presents in the second decade of life, but should be avoidable with good cardiac follow-up and surgery to close the defect if required.

105
Q

A child with known complex congenital heart disease develops a fever.

Which of the following is NOT a manifestation of infective endocarditis?

Microscopic haematuria
Sydenham chorea
Small necrotic skin lesions
Splenomegaly
Acute hemiplegia
A

Sydenham chorea

Sydenham chorea (rapid jerking movements mainly of the face, feet and hands.) is a feature of rheumatic fever.

Clinical signs of infective endocarditis include:
• Fever
• Anaemia and pallor
• Splinter haemorrhages in nailbed
• Clubbing (late)
• Necrotic skin lesions
• Changing cardiac signs
• Splenomegaly
• Neurological signs from cerebral infarction
• Retinal infarcts
• Arthritis/arthralgia
• Haematuria (microscopic).
106
Q

A congenital heart defect is suspected on an antenatal ultrasound scan.

Which of the following is the most common type of congenital heart disease in the UK?

Atrial septal defect
Persistent arterial duct
Pulmonary stenosis
Tetralogy of Fallot
Ventricular septal defect
A

VSD

This is the most common single group of structural congenital heart disease (30%).

107
Q

Sunil, a 3-month-old boy, presents with breathlessness and sweating on feeding. He has had several chest infections. You suspect heart failure.

Which of the following is true regarding his heart failure?

His heart failure may be caused by Eisenmenger syndrome
Hepatomegaly is not a feature at this age
It is most likely to be due to left heart obstruction
Symptoms from a large left-to-right shunt are likely to become more severe after 3 months of age due to a rise in pulmonary vascular resistance
it is most likely due to a left-to-right shunt

A

it is most likely due to a left-to-right shunt

After the 1st week of life, progressive heart failure is most likely due to a left-to-right shunt, most often from a ventricular septal defect.

Symptoms of heart failure are likely to increase up to about 3 months of age because of the fall in pulmonary vascular resistance. However, symptoms may subsequently become less severe because of a rise in pulmonary vascular resistance.

108
Q

You see the child of a refugee from Pakistan. In the history you are told that he has had significant fever and joint pain on several occasions. You suspect rheumatic fever.

Which of the following is a major criterion of rheumatic heart disease?

Chorea
Fever
Polyarthralgia
Raised acute-phase reactant, e.g. CRP
Prolonged PR interval on ECG
A

Chorea

Rheumatic fever is now rare in the developed world, but remains the most important cause of heart disease in children worldwide.
Major criteria are:
• Chorea (rapid, jerky movements mainly of face, hands and feet)
• Pancarditis
• Polyarthritis
• Erythema marginatum
• Subcutaneous nodules.
The diagnosis requires 2 major or 1 major and 2 minor criteria, plus evidence of preceding Group A streptococcal infection.

109
Q

Ben, aged 2 months, has developed difficulty breathing following an upper respiratory tract infection. On examination his temperature is 38°C, his respiratory rate is 60 breaths/minute and heart rate of 180 beats/minute. His tongue is pink. On auscultation there is a thrill, a loud (Grade 4/6) pansystolic murmur at the left sternal edge and the liver is palpable 4 cm below the costal margin.

Select the most likely diagnosis.

Cyanotic congenital heart disease
Heart failure
Aortic stenosis
Atrial septal defect (ASD)
Coarctation of the aorta
Dextrocardia
Mitral regurgitation
Mitral stenosis
Normal
Patent ductus arteriosus (PDA)
Ventricular septal defect (VSD)
A

Heart failure, secondary to ventricular septal defect.

110
Q

Geoffrey is a 31-weeks’ gestation Zambian neonate. On the neonatal intensive care unit, he has his first newborn examination. The junior doctor notices that his left testis is undescended.
Which of the following statements about undescended testis is correct?

Undescended testes are normal in an infant less than 32 weeks’ gestation

Undescended testes should be corrected at any time after the age of 2 years

A retractile testis is likely to require an orchidopexy

An undescended abdominal testis can be identified on deep abdominal palpation

Impalpable testis location needs to be ascertained for cosmetic reasons

A

Undescended testes are normal in an infant less than 32 weeks’ gestation

Testes usually descend sometime in the third trimester. His testis should be checked at discharge.

111
Q

Khalid, a 9-month-old Asian boy, is brought to his family doctor by his mother. She would like him to have a circumcision. However, routine circumcision is not medically provided in the UK. Which of the following would be a medical indication for circumcision?

Hypospadias
Paraphimosis
Non-retractile foreskin in a one-year-old boy
A urinary tract infection in a boy under 1 year of age
Recurrent attacks of balanoposthitis

A

Recurrent attacks of balanoposthitis

Balanoposthitis is treatable with antibiotics, but if it is recurrent, circumcision may be indicated.

112
Q

Manuel, a 3-year-old Spanish boy, had a Kasai procedure for biliary atresia in infancy. Unfortunately, he now presents with chronic liver disease.

Which of the following signs would you NOT expect to see in Manuel?

Bruising
Clubbing
Splenomegaly
Erythema multiforme
Encephalopathy
A

Erythema multiforme

Liver disease does not cause erythema multiforme. Erythema multiforme is a rash with ‘target lesions’: a central papule surrounded by an erythematous ring. Causes include herpes, mycoplasma or other infections, or drug reactions, but not liver failure.

113
Q

Reece, a 4-week-old Caucasian boy living in the UK, is taken to his family doctor because he is jaundiced. He was born at term and is breast-fed. His mother reports that he has always looked yellow, and has started to develop bruises. His stools are now pale in colour. On examination he has hepatomegaly.

Which of the following is the most important diagnosis to rule out?

Breast-milk jaundice
Congenital infection
Hypothyroidism
Cystic fibrosis
Biliary atresia
A

Biliary atresia

No infant with jaundice should ever be ignored or given false reassurance if they are over 2 weeks of age. The infant should be assessed to ensure they are clinically well and their stool and urine should be inspected (pale stools strongly suggest bile flow obstruction). The level of conjugated bilirubin should be measured. Biliary atresia presents with conjugated jaundice and needs to be operated on as soon as possible to reduce the risk of the infant requiring a liver transplant.

114
Q

Stacey, a 14-year-old Caucasian girl, took an overdose of 30–40 paracetamol tablets 2 days ago, washed down with a quantity of ‘Alco-pop’. She presents with right upper quadrant abdominal pain. Investigations reveal a rise in her bilirubin and liver transaminases.

Which of the following statements about the management of her acute liver failure is FALSE?

Plasma albumin is the best indicator of Stacey’s liver function
Hypoglycaemia is a recognised complication
She requires evaluation of her mental state
Prothrombin time is a useful prognostic marker
Stacey should be given vitamin K

A

Plasma albumin is the best indicator of Stacey’s liver function

A fall in plasma albumin levels is a late marker of liver disease.

115
Q

Javid is a 2-month-old Asian baby born at term in Pakistan. He presents with jaundice. His mother’s blood group is AB rhesus positive. His stool and urine are a normal colour. He is breast-fed, although he has not been feeding well. His mother is concerned that this could be due to his constipation. When you examine the child you note that he has dry skin and an umbilical hernia.

Which of the following is the most likely diagnosis?

Biliary atresia
Congenital infection
Galactosaemia
Haemolytic anaemia
Hypothyroidism
A

Hypothyroidism

Hypothyroidism is a cause of prolonged jaundice in infants. Clinical features include dry skin, constipation, coarse facial features, umbilical hernia and a hoarse cry. In the UK it is usually identified on biochemical screening (Guthrie test).

116
Q

Lun, a 3-year-old Cantonese boy, has hepatitis B. He is asymptomatic, and the hepatitis B infection was picked up on a routine blood test.

Which of the following is NOT a transmission route for hepatitis B?

Perinatal transmission from carrier mother
Transfusion of infected blood or blood products
Needlestick injuries with infected blood
Horizontal spread within families
Transmission via infected faecal material

A

Transmission via infected faecal material

Faeco–oral transmission occurs with hepatitis A and hepatitis E, but not with hepatitis B.

117
Q

Summer, a 12-year-old Caucasian girl, is seen in the Accident and Emergency department. Her parents report that her school performance has been deteriorating and recently she has become confused and unsteady on her feet.
Examination findings of her eyes show Kayser-Fleischer rings:

Select the most likely diagnosis.

α1-Antitryspin deficiency
Autoimmune hepatitis
Bacterial infection
Biliary atresia
Cystic fibrosis
Hepatitis A
Hepatitis B
Hepatitis C
Inflammatory bowel disease
Galactosaemia (metabolic disease)
Primary sclerosing cholangitis
Wilson disease
A

Wilsons

This is a rare autosomal recessive condition. The clinical sign shown in the picture is a Kayser–Fleischer ring. The basic genetic defect is a combination of reduced synthesis of caeruloplasmin (the copper-binding protein) and defective excretion of copper in the bile, which leads to an accumulation of copper in the liver, brain, kidney and cornea.

Neuropsychiatric features are more common in those presenting from the second decade onwards and include deterioration in school performance, mood and behaviour change, and extrapyramidal signs such as incoordination, tremor and dysarthria.

Other diagnoses are incorrect, because Kayser–Fleischer rings are a feature of Wilson disease only.

118
Q

Ahmed is a 5-year-old boy from Pakistan who is transferred to a paediatric oncology centre as he is diagnosed with malignant disease.

Statistically, what is the most likely cause of his malignancy?

Leukaemia
Renal (Wilms) tumour
Neuroblastoma
Brain tumour
Bone tumour
A

Leukaemia

Leukaemia is the commonest malignancy in childhood, and in a child of his age acute lymphoblastic leukaemia (ALL) is most likely. It accounts for 80% of leukaemia in children.

119
Q

Polly is a 2-year-old girl who is receiving chemotherapy for her acute lymphoblastic leukaemia (ALL). She is known to be neutropenic and has developed a fever > 38.5 °C.

What is the most appropriate course of action?

Be admitted to hospital for observation
Be started immediately on oral antibiotics
Have her blood count and inflammatory markers (e.g. CRP) measured
Be assessed in hospital, have blood cultures taken and started on IV antibiotics
See her general practitioner for further assessment and decision regarding antibiotics

A

Be assessed in hospital, have blood cultures taken and started on IV antibiotics

Children with fever and neutropenia are at high risk of serious infection and must be admitted promptly to hospital for cultures and treatment with broad-spectrum antibiotics.
Merely observing or just giving oral antibiotics would not be aggressive enough for what could be a life-threatening infection.

120
Q

Mohammed is a 3-year-old boy who is reviewed in the paediatric assessment area. He has a history of weight loss and lethargy. His mother is also concerned as he keeps crying and complains of pain in his tummy. On examination of his abdomen he has an extensive mass. The doctor is worried he may have a neuroblastoma.

Which of the following investigations would be the most useful in making the diagnosis?

Urinary catecholamines
Serum HCG (human chorionic gonadotrophin)
Plasma LDH (lactate dehydrogenase)
Plasma ammonia
Serum αFP (αfetoprotein)
A

Urinary catecholamines

Characteristic clinical and radiological features with raised urinary catecholamine levels suggest neuroblastoma.
Confirmation is usually by biopsy. Evidence of metastatic disease is detected with bone marrow sampling and MIBG (metaiodobenzyl-guanidine) scan with or without a bone scan.

121
Q

Josh is a 2-year-old boy who presents to the emergency department with weight loss and lethargy. On examination he has splenomegaly. You are worried that he has leukaemia. You perform an urgent full blood count.

What are the most likely haematology results if he has leukaemia?

Low haemoglobin and low platelets
High haemoglobin and low platelets
Normal haemoglobin and normal platelets
High haemoglobin and raised platelets
Low haemoglobin and raised platelets
A

Low haemoglobin and low platelets

A low haemoglobin and low platelet count is often present at diagnosis due to bone marrow failure.
A blood film should be performed for evidence of circulating leukaemic blast cells.

Bone marrow examination is essential to confirm the diagnosis and to identify immunological and cytogenetic characteristics which give useful prognostic information. A chest X-ray is required to identify a mediastinal mass characteristic of T-cell disease.

122
Q

Mark, a 3-year-old boy, is currently receiving chemotherapy. His sister has developed chickenpox. Mark is well and does not have a fever.

Mark’s parents are worried and phone the oncology ward for advice.

What information should they be given?

Reassurance – this is a common illness that most children get
Monitor Mark and if he has any signs of the rash bring him to the ward
Mark needs urgent treatment to prevent him from becoming unwell
Advise them to see their general practitioner to check that he is all right
Monitor Mark and if he becomes unwell with a fever, bring him to the ward

A

Mark needs urgent treatment to prevent him from becoming unwell

His sister has varicella zoster (chickenpox) infection, and this can be life-threatening in immunocompromised patients. He needs treatment with VZIG (varicella zoster immunoglobulin) unless he is immune. The incubation period for varicella is 14–21 days, so one cannot be reassured by the fact that he is currently well.

123
Q

Essa is a 2-year-old boy from Pakistan who has recently developed a squint. He has no other past medical history and is not currently on any medication. On examination the movement of the eyes are normal and the pupils are equal and reactive to light. Whilst checking his pupillary reflex you notice his left pupil looks red but the right looks white. His systemic examination is normal.

What is the likely cause of this?

VIth nerve palsy
Retinoblastoma
Trauma
Brain tumour
Foreign body
A

Retinoblastoma

White papillary reflex (also known as leukokoria) requires urgent ophthalmological assessment as it can be caused by:
- retinoblastoma
- corneal opacity
- congenital cataract (usually presents shortly after birth)
- vitreous opacity
- retinal disease e.g. retinal detachment.
There is no history of trauma or a foreign body making them unlikely.

124
Q

Tia is a 4-year-old Caucasian girl. She is referred to the paediatric ward by her general practitioner as her mother noted that she had a yellow tinge to her eyes since developing an upper respiratory tract infection. She is well in herself and has no history of weight loss. There is no family history of any blood disorders. On examination, she was pale and her spleen was enlarged 3 cm below the costal margin.

You perform a full blood count which reveals:
Hb 6 g/dl, WBC 8 ×109/L
Platelets 255×109/L
Blood film – small red cells

What is the most likely diagnosis?

Acute lymphoblastic leukaemia (ALL)
Glucose-6-phosphate dehydrogenase (G6PD) deficiency
Hereditary spherocytosis
Sickle cell disease
Thalassaemia
A

Hereditary spherocytosis.

In hereditary spherocytosis the anaemia is usually mild (haemoglobin 9–11 g/dl), but the haemoglobin level may transiently fall during infections. Mild to moderate splenomegaly is common.

125
Q

Marco, whose parents come from Spain, is a 10-year-old boy who presents to his general practitioner. This evening he is more lethargic than usual and his urine has become dark in colour. There is no history of excessive exercise or beetroot consumption, in fact they had broad beans for lunch. His examination is normal and he is afebrile. He has no past medical history of note. He has not had a recent upper respiratory tract infection.

What is the underlying cause of his new symptoms?

Sickle cell disease
β-thalassaemia trait
Acute lymphatic leukaemia
Glucose-6-phosphate dehydrogenase (G6PD) deficiency
Pyelonephritis
A

Glucose-6-phosphate dehydrogenase (G6PD) deficiency

In G6PD deficiency, acute haemolysis may be precipitated by infection, the most common precipitating factor. Certain drugs including nitrofurantoin, fava beans (broad beans) and naphthalene in mothballs can also precipitate haemolysis. The urine is dark as it contains haemoglobin as well as urobilinogen.

126
Q

A mother knows that she is a sickle cell carrier and wants to know if there is a chance her child will be affected. She attends the genetics clinic at the hospital.

How is sickle cell disease inherited?
Autosomal dominant
Autosomal recessive
X-linked dominant
X-linked recessive
Imprinting
A

Autosomal recessive

If the mother is a carrier, her partner would also need to be a carrier for her unborn child to be affected. Standard practice in the UK is for the partners of all sickle carriers to be screened. All newborn babies are screened for sickle cell disease irrespective of parental carrier status.

127
Q

Tom is a 5-year-old boy who presents to hospital with a recent history of bruising easily. 2 weeks ago he had an upper respiratory tract infection which resolved spontaneously. On examination today he is afebrile and feels well, but has lots of bruises over his bony prominences and around his inner thighs, with some scattered petichae.

Investigations reveal:
• Hb 11.6 g/dl, WBC 10.2 × 109/L, platelet count 50 × 109/L
• PT 16 sec (control 12–15 sec)
• APTT 30 sec (control 25–35 sec)
• Fibrinogen 2.5 g/L (normal 2–4 g/L)

What is the most likely diagnosis?

Acute lymphoblastic leukaemia (ALL)
Non-accidental injury
Haemophilia A
Immune thrombocytopenia (ITP)
Vitamin D deficiency
A

Immune thrombocytopenia (ITP)

Affected children develop petechiae, purpura and/or superficial bruising.

128
Q

Amir is a 2-year-old Bangladeshi boy who has eczema. His general practitioner is undertaking a routine review of his care and notices that Amir is pale. The remainder of his examination is normal. His mother reports that he has a poor diet.

Investigations reveal:
• Hb 6.6 g/dl, WBC 10.2 × 109/L, platelet count 350 × 109/L
• MCV 60 fl

What is the most appropriate treatment?

Dietary advice
Folic acid
Vitamin B12 injections
Iron supplements
Multivitamin tablet
A

Iron supplements

Iron deficiency anaemia is the most likely diagnosis and iron therapy is indicated.
In some children anaemia can be due to a beta-thalassaemia trait. The blood parameters should be repeated after a course of treatment and if there has not been a response he should be tested for beta-thalassaemia trait.

129
Q

Joseph, a 2-year-old Afro-Caribbean boy from London is admitted to hospital for an elective repair of an inguinal hernia. He has no other medical problems. His pre-operative assessment reveals the following results and blood film.

Hb = 8.6 g/dl
MCV = 68 fl (normal 75–87 fl)
MCHC = 22 g/dl (normal 32–35 g/dl)
WBC = 11.2×109/L
Platelets = 262×109/L
Hb electrophoresis = HbA 98%, HbA2 2%
ITP
Haemophilia A
Vitamin K deficiency
von Willebrand Disease (vWD)
Liver disease
Meningococcal disease
ALL
Acute myeloid leukaemia
Vitamin D deficiency
G6PD deficiency
Infectious mononucleosis
Iron deficiency anaemia
Sickle cell disease
Normal variant (no pathology present)
β-thalassaemia major
β -thalassaemia trait
α-thalassaemia major
α-thalassaemia trait
A

Iron deficiency anaemia

It could be α-thalassaemia trait, which should be ruled out if there is no response to iron therapy.

130
Q

Amber from Kenya, aged 9 months, presents to the Accident and Emergency department. She has a 6-hour history of pain in her fingers. She has had an upper respiratory tract infection for the past 24 hours. She has no other past medical history and is not on any medication.

ITP
Haemophilia A
Vitamin K deficiency
von Willebrand Disease (vWD)
Liver disease
Meningococcal disease
ALL
Acute myeloid leukaemia
Vitamin D deficiency
G6PD deficiency
Infectious mononucleosis
Iron deficiency anaemia
Sickle cell disease
Normal variant (no pathology present)
β-thalassaemia major
β -thalassaemia trait
α-thalassaemia major
α-thalassaemia trait
A

Sickle cell disease

She is experiencing a vaso-occlusive crisis. A common mode of presentation in late infancy is the hand–foot syndrome, in which there is dactylitis, with swelling and pain of the fingers and/or feet.

131
Q

James is a 3½-year-old boy who wets the bed about 4 out of every 7 nights. He doesn’t wake after wetting. He has been dry during the day for 1 year. There is no history of constipation. He is otherwise well, and has a normal examination. What would be the most appropriate initial management?

Explanation and advice to parents
Enuresis alarm
Desmopressin therapy
‘Lifting’ at midnight
Urology referral
A

Explanation and advice to parents

Nocturnal enuresis is common at his age.

132
Q

Zoe is a 14-year-old girl who has always been quiet and hard working. She is very thin and is losing weight despite being very interested in food; she enjoys baking cakes for her classmates and is always keen to go to the supermarket with her mother. Zoe herself denies that there is a problem with her weight or that she has been vomiting. Her parents are both extremely worried about her weight loss and poor appetite; they have not noticed any vomiting. There are no other symptoms and her examination is normal except that her height is on the 30th centile but her weight is below the 0.4th centile.

What is the most likely diagnosis?

Depression
Substance misuse
Anorexia nervosa
Bulimia
Psychosis
A

Anorexia nervosa

Zoe has several characteristic features—female, adolescent, weight below the 0.4th centile, but has a distorted body image in that she denies there is a problem with her weight. She has perfectionist personality traits, and is overly interested in food without eating it herself.

133
Q

Florence is a 7-year-old girl whose mother brings her to her general practitioner. Her mother reports that her daughter is hyperactive, and that she is ‘at the end of her tether’. Florence has always been a very active girl since learning to walk at 11 months of age. Though being described as an ‘angel’ at school she is very difficult to manage at home. She is always on the go. She has tantrums when not getting her own way. She will often move from task to task at home without completing one before going on to the next. She has never been in any accidents and is not impulsive. Florence is currently staying with her grandparents 2 nights a week to give her mother a break. She has no other medical problems and is not on any medication. There is no family history of similar problems.

Which of the following would be the first step in management of this girl?

Psychostimulant e.g. methylphenidate
Sedative medication
Referral to hospital paediatric department
Parenting classes
Referral to child psychiatrist
A

Parenting classes

Often effective in managing such problems. Many children are incorrectly labelled by family members or teachers as having Attention Deficit Hyperactivity Disorder (ADHD). In this case her difficult behaviour is not pervasive, as it is not present at school.

134
Q

Georgio is a 6-year-old, normally healthy boy who attends the paediatric assessment unit. 3 weeks ago he had an episode of diarrhoeal illness lasting 3 days, which was not associated with vomiting and resolved without any treatment. Now he is having episodes of cramping abdominal pains. He has not opened his bowels for a week. He reports that his stools were normal prior to the diarrhoeal illness. He is his normal self between the episodes of abdominal pain. On examination he is a well child, not currently in pain, with an indentable mass in his left iliac fossa. His weight and height are plotted and are found to lie on the 25th centile.

What is the most likely diagnosis?

Constipation
Malrotation with volvulus
Recurrent abdominal pain of childhood
Appendicitis
Intussusception
A

Constipation

135
Q

Jeremy, aged 4 years, is brought to his general practitioner because he wakes some nights at about 10 pm with a shout. When his parents go to him he looks awake but confused. He goes back to sleep soon afterwards and next morning cannot recall any of these night-time events. He is otherwise well and has no medical problems. During the day he attends nursery and is thriving in this environment.

What is the most likely explanation?

Nightmares
Night terrors
Complex partial seizures
Motor tics
Psychotic behaviour
A

Night terrors

136
Q

Sharon, aged 15 years and from the UK, is admitted to hospital following an overdose of ten 500 mg paracetamol tablets. She took all these tablets at the same time along with a glass of wine stolen from her parents’ cabinet. Paracetamol levels are taken and treatment is not required. She makes an uneventful recovery on the paediatric ward. A more detailed history reveals that she has not been feeling herself for the past few months. She has had chest and stomach pains. Her mother relates that Sharon has been feeling down and uninterested in life. Her teachers are very concerned about the deterioration in her school performance.

Select your discharge management plan.

Reassure that this is a normal part of adolescence
Design a rehabilitation programme with her parents
Refer to a child and adolescent mental health team
Perform a cranial CT scan
Arrange general practitioner follow-up

A

Refer to CAMHS

Sharon should have an assessment of her mental health prior to being discharged from the ward. This is to ensure that there is not an ongoing suicide risk, and to introduce her to therapeutic services.

137
Q

For the last 3 months Felicity, aged 13 years and from the UK, has complained of difficulty in doing school work, increased tiredness on the slightest exercise, pain in her joints and headaches with tenderness over the top of her head. This began shortly after a febrile illness, associated with a cough and sore throat, which lasted 3 days. She has not attended school for the last 8 weeks. Examination is normal. You plot her height and weight which are on the 30th and 9th centile respectively. There are no other medical problems and she is not on any medications.

On the basis of this history alone which of the following is the most likely diagnosis?
Tuberculosis
Brain tumour
Depression
Hypothyroidism
Chronic fatigue syndrome
A

Chronic fatigue syndrome

138
Q

For the last 3 months Felicity, aged 13 years and from the UK, has complained of difficulty in doing school work, increased tiredness on the slightest exercise, pain in her joints and headaches with tenderness over the top of her head. This began shortly after a febrile illness, associated with a cough and sore throat, which lasted 3 days. She has not attended school for the last 8 weeks. Examination is normal. You plot her height and weight which are on the 30th and 9th centile respectively. There are no other medical problems and she is not on any medications. You suspect chronic fatigue syndrome.

What would be the most appropriate management plan?

Complete bed rest
Home tuition
Gradual rehabilitation programme
Intensive exercise
Neurostimulant medication
A

Gradual rehabilitation programme

Usually the most successful approach in chronic fatigue syndrome. It consists of a gradual increase in exercise, school work and activities of daily living.

139
Q

William is an 8-year-old boy who presents to the Accident and Emergency department with a fever and arthralgia. On examination, he is noticed to have a painful, raised rash on his shins.

Which of the following is the most likely cause?

Streptococcal infection
Herpes simplex infection
Mycoplasma pneumoniae infection
Henoch–Schönlein purpura
Football injuries
A

Streptococcal infection

This boy has erythema nodosum and the causes include streptococcal infection, primary tuberculosis, inflammatory bowel disease, drug reaction or idiopathic.

140
Q

Many rashes in childhood are itchy and can cause a lot of discomfort for the child.

Which of the following rashes is not itchy?

Infantile seborrhoeic dermatitis
Chickenpox
Scabies
Candida napkin rash
Pityriasis rosea
A

Infantile seborrhoeic dermatitis

The causes of itchy rashes are: atopic eczema, chickenpox, urticaria, allergic reactions, contact dermatitis, insect bites, scabies, fungal infections and pityriasis rosea. All these conditions are itchy, other than seborrhoeic dermatitis.

141
Q

You see a 10-month-old baby boy who has been diagnosed with atopic eczema. His parents are struggling to control it. He is scratching himself all day and at night, and appears in discomfort.

Which of the following is useful advice?

Apply emollients only once a day
Regularly wash the baby with soap to avoid infection
Using nylon instead of cotton clothes
Use ointments instead of creams when the skin is dry
Bandages, though helpful, can only be used in children over 1 year of age

A

Use ointments instead of creams when the skin is dry

There are many different management options for eczema. Ointments are preferable to creams when the skin is dry. Topical cortiocsteroids, immunomodulators and antibiotics are used for flair ups.

142
Q

Mikey is a 2-year-old African boy who has known eczema. He comes to his family doctor with his mother, as his eczema has become troublesome over the last few weeks. His mother had been using emollients twice a day, but for the last few months during the summer she stopped as she did not think his eczema had been sufficiently bad. Today he has erythematous, weeping areas of skin in the flexor surfaces of his knees and ankles. There are some areas of yellow crusting and he is pyrexial.

Which of the following could have caused his eczema to flare up?

Bacterial infection
Increase in heat due to summer
Reduction in his emollients
Viral infection
Exposure to allergen
A

Bacterial infection

Causes of flare up of eczema are:
• Bacterial infection, e.g. Staphylococcus, Streptococcus spp.
• Viral infectino, e.g. herpes simplex virus
• Ingestion of an allergen, e.g. egg
• Contact with an irritant or allergen
• Environment: heat, humidity
• Change or reduction in medication
• Psychological stress
The yellow crusting and pyrexia is suggestive of infection with Staphylococcus, therefore a bacterial cause for the exacerbation.

143
Q

Ellie, a 7-year-old girl, is newly diagnosed with diabetes mellitus. She has been drinking lots of fluid and has had to pass urine frequently. She has a markedly raised blood glucose and heavy glycosuria.

Which of the following is true about her diabetes?

The incidence in the UK is falling
There is autoimmune pancreatic β-cell damage
She will have gained weight in the last few weeks
Her diagnosis should be confirmed with a glucose tolerance test
She can be managed with oral hypoglycaemic agents and dietary modification

A

There is autoimmune pancreatic β-cell damage

144
Q

James, aged 11, has type 1 diabetes mellitus. While playing football during the mid-morning break at a holiday camp, he suddenly feels faint. His classmates call the supervisor who finds him lying unresponsive in the playground. The supervisor is aware that James has diabetes.

What should be his immediate management?

Check blood glucose
Give insulin
Call an ambulance
Give buccal glucose gel
Give a glucose drink
A

Give buccal glucose gel

He is hypoglycaemic brought on by exercise. Glucose is absorbed quickly from buccal glucose gel, and he should rapidly wake up. He will need to have a carbohydrate-containing drink and snack to maintain his blood glucose.

145
Q

Mohammed, a 12-year-old boy with type 1 diabetes mellitus, is reviewed in the outpatient clinic. In spite of good control of his diabetes, he is losing weight and is not gaining height as rapidly as his peers. He has lost interest in sport as he says he can’t keep up with the other boys any more. His appetite is not as good as in the past. Other than his growth failure, there are no other abnormalities on examination.

What is the most likely diagnosis?

Hypothyroidism
Depression
Inflammatory bowel disease
Anorexia nervosa
Growth hormone deficiency
A

Hypothyroidism

Children with type 1 diabetes mellitus are at increased risk of thyroid disease and should be screened annually. They are also at increased risk of coeliac disease.

146
Q

Zeinab, aged 15 years, has had 3 months of diarrhoea, weight loss and palpitations.
You suspect hyperthyroidism.

How would you confirm the diagnosis?

High TSH and high T4 levels
High TSH and low T4 levels
Low TSH and high T4 levels
Low TSH and Low T4 levels
Normal TSH and high T4 levels
A

Low TSH and high T4 levels

The levels of thyroxine (T4) and/or tri-iodothyronine (T3) are elevated and TSH levels are suppressed to very low levels in hyperthyroidism.

147
Q

Lara, an 18-day-old girl, has congenital adrenal hypoplasia. She presents having vomiting all her feeds today, and has become lethargic and will no longer feed. On examination she is tachycardic, with a heart rate of 160 beats/minute and a capillary refill time of 3 seconds. A bedside blood test shows that she is hypoglycaemic.

What is the immediate management?

IV saline, dextrose and hydrocortisone
Oral rehydration
Oral rehydration and oral glucocorticoid
IV dextrose alone for 24 hours
Oral rehydration, sodium supplements and oral glucocorticoid
A

IV saline, dextrose and hydrocortisone

Lara is having an adrenal crisis. This requires urgent treatment with intravenous saline, glucose and hydrocortisone. Long-term treatment is with glucocorticoid and mineralocorticoid replacement and oral sodium chloride may be needed during infancy. The dose of glucocorticoid needs to be increased at times of illness.

148
Q

A newborn baby is diagnosed with a metabolic condition at 10 days of age, after she presented with vomiting, jaundice, hepatomegaly and liver failure. She was subsequently put on a lactose- and galactose-free diet.

Which enzyme is deficient?

Galactose-1-phosphate uridyl transferase
Glucose-6-phosphatase
Glucose-6- phosphate dehydrogenase (G6PD)
Galactokinase
Phosphoglucomutase
A

Galactose-1-phosphate uridyl transferase

This child has galactosaemia, which is a rare disorder resulting from deficiency of the enzyme galactose-1-phosphate uridyl transferase, which is essential for galactose metabolism. When lactose-containing milk feeds such as breast-milk or infant formula are introduced, affected infants feed poorly, vomit and develop jaundice and hepatomegaly and hepatic failure. Management is with a lactose- and galactose-free diet for life.

149
Q

Terry was born at term and was well at birth. On day 2 of life he became severely unwell with poor feeding, vomiting, acidosis and encephalopathy. Sepsis and hypoglycaemia were excluded, and an inborn error of metabolism is thought to be the cause.

Which of the following is the most likely cause?

Urea cycle defect
Mucopolysaccharide disease
Medium-chain acyl-CoA dehydrogenase deficiency (MCAD)
Glycogen storage disease
Fatty acid oxidation defect
A

Urea cycle defect

Inborn errors of metabolism present in different ways. Urea cycle or organic acid disorders: presentation as described for Terry.

150
Q

Luke, a 4-month-old boy, presents with a history of vomiting and diarrhoea. Today he has vomited numerous times, and has become lethargic. On examination he is tachycardic, with a heart rate of 160 beats/minute; his capillary refill time is 2–3 seconds. A bedside blood test showed that he was hypoglycaemic. A diagnosis of Addison disease is considered.

Which of the following biochemical results would you expect with this diagnosis?

Hypernatraemia and hyperkalaemia with low cortisol
Hypernatraemia and hypokalaemia with high cortisol
Hyponatraemia and hypokalaemia with low cortisol
Hyponatraemia and hyperkalaemia with high cortisol
Hyponatraemia and hyperkalaemia with low cortisol

A

Hyponatraemia and hyperkalaemia with low cortisol

In Addison disease there is hyponatraemia, hyperkalaemia and hypoglycaemia. The plasma cortisol is low and the plasma ACTH concentration high.

151
Q

Amir is a 5-year-old boy who was born and lives in the UK. His parents are from Pakistan. He presents to the emergency department with a high temperature and complaining of pain in his right upper arm. This has been present for 2 days and seems to be getting worse. The pain is described as being sharp, and gets worse when he moves his arm. He had an abscess on his right finger 2 weeks ago for which he was prescribed oral antibiotics. On examination he has a temperature of 39°C. There is an area of redness and swelling over the right upper arm, which is painful on touching. There is pain on moving the arm. You perform a blood test including a full blood count, which reveals a raised white cell count and a C-reactive protein which is raised.

What is the most likely causative organism of his infection?

Haemophilius influenza
Staphylococcus aureus
Streptococcus viridans
Salmonella typhi
Mycobacterium tuberculosis
A

Staphylococcus aureus

Amir is most likely to have osteomylelitis. Most infections are caused by Staphylococcus aureus, but other pathogens include Streptococcus. In sickle cell anaemia, there is an increased risk of staphylococcal and salmonella osteomyelitis.

152
Q

Aisha, a 14-year-old girl from Bangladesh, has developed a right-sided limp that has been present for 2 weeks. She is now complaining of pain in her right hip and knee. On examination, she has a temperature of 37.7°C. She is overweight, with a BMI of 26. She has a decreased range of movement of the right hip. Blood tests, including a full-blood count and a C-reactive protein, are taken and are found to be normal.

What is the most likely diagnosis?

Osteomyelitis
Osgood–Schlatter disease
Perthes disease
Septic athritis
Slipped capital femoral epiphysis
A

SUFE

This girl has a slipped capital femoral epiphysis (SCFE; also known as slipped upper femoral epiphysis [SUFE]). This results in displacement of the epiphysis of the femoral head postero-inferiorly. It is most common at 10–15 years of age during the adolescent growth spurt, particularly in obese boys.

153
Q

Zain is a 14-year-old boy who presents to the emergency department. He is a good footballer, and after training today he complained of pain in his left knee. He has had the pain for a while, and it gets worse when he exercises. On examination there is swelling over the left tibial tuberosity. He is afebrile. There is no night pain or pain on waking.

What is the most likely diagnosis?

Osteomyelitis
Osgood–Schlatter disease
Perthes disease
Septic arthritis
Slipped capital femoral epiphysis
A

Osgood–Schlatter disease

This is osteochondritis of the patellar tendon insertion at the knee, often affecting adolescent males who are physically active (particularly football or basketball). It usually presents with knee pain after exercise, localised tenderness and sometimes swelling over the tibial tuberosity. There is often hamstring tightness. It is bilateral in 25–50%.

154
Q

Edward, aged 3 years, woke up complaining of pain in his right leg. Yesterday he refused to play in the garden with his older brother, preferring to watch television. Since then he has become more unwell and his mother reports he is more lethargic than usual. On examination he has a temperature of 39°C, a heart rate of 180 beats/minute and a respiratory rate of 25 breaths/minute. His right knee is red and swollen and he cries if it is extended or flexed. On further examination you note a small healing insect bite on his right ankle. He has no other medical problems and is not currently on any medications.

What is the most likely diagnosis?

Osteomyelitis
Septic arthritis
Henoch–Schönlein purpura arthritis
Trauma
Juvenile idiopathic arthritis (JIA)
A

Septic arthritis

The main differential diagnosis would be osteomylelitis. In this case the joint itself is red and swollen, with pain on movement of the joint, making septic arthritis more likely. An ultrasound and aspiration of the joint should be performed to confirm this.

155
Q

Karen, aged 5 years, presents to her general practitioner. She has been unwell for 2 weeks with lethargy, fever and painful wrists and knees. On examination she has a temperature of 38.5°C and a subtle erythematous rash on her trunk. You identify that several joints are swollen, warm and painful to move, namely the wrists, her right elbow, the knees, her left ankle and left hip. She has some cervical lymphadenopathy and her spleen is palpable. She has no previous medical problems and her mother has been giving paracetamol. You perform some blood tests and get the following results:

  • Hb 8.5 g/dl, WBC 17 × 109/L, neutrophils 10.4 × 109/L, platelets 366 × 109/L
  • Blood film normal, no atypical lymphocytes
  • Erythrocyte sedimentation rate (ESR) 70 mm/hr
  • ANA (Anti-nuclear antibody) –ve
  • Double-stranded DNA –ve
  • ASO titre normal

What is the most likely diagnosis?

Acute lymphoblastic leukaemia (ALL)
Systemic lupus erythematosus (SLE)
Epstein Barr virus (EBV) infection
Post-streptococcal arthritis
Systemic onset juvenile idiopathic arthritis (JIA)
A

Systemic onset JIA

Is the most likely diagnosis, as she has a systemic illness, polyarthritis (more than four joins) and a salmon-pink rash. The raised neutrophil count and ESR are consistent with the diagnosis.

156
Q

William is an 8-year-old boy from London who presents with joint pain. His mother reports that since yesterday he has been refusing to walk, as his legs are so painful. His father has been carrying him in order for him to get around. They report that the pain is getting worse and paracetamol does not seem to help. He has no other complaints and has not had a fever at home. On examination his temperature is 37 °C and his heart rate is 100 beats/minute. He is settled at rest and playing a computer game on his iPad. On further examination you notice a rash over his lower limbs, buttocks and elbows. The rash comprises some large (5–15 mm), red, raised lesions and multiple pin-point lesions which do not blanche on pressure. His peripheries are warm and he has a good pulse volume with a normal capillary refill time. There is some generalised swelling around his knees and ankles. He has pain on passive movement of both knees and ankles.

What is the most likely diagnosis?

Post-streptococcal glomerulonephritis
Reactive arthritis
Systemic onset juvenile idiopathic arthritis (JIA)
Meningococcal septicaemia
Henoch–Schönlein purpura (HSP)
A

HSP

HSP is the most common vasculitis of childhood, and presents with a purpuric rash over the lower legs and buttocks, often associated with arthritis of the ankles or knees. In this case the child presented with the arthritis rather than the rash. Other features are abdominal pain, haematuria and proteinuria.

157
Q

Dominika is a 3-year-old girl from the Ukraine. She presents to the emergency department with an acute onset limp, which was not present on the previous day. Her mother reports that she was unwell 2 weeks ago with a coryzal illness. The pain is in her right leg and is present only on walking. On examination she has a temperature of 37 °C. There is a limp of her right leg on walking. On examination the hip and leg look normal, but on passive movements of her right hip, there is decreased external rotation.

What is the most likely diagnosis from the list below?

Perthes disease
Septic arthritis
Slipped capital femoral epiphysis
Transient synovitis
Bone tumour
A

Transient synovitis

This is the most common cause of acute hip pain in children. It occurs in children aged 2–12 years of age. It often follows or is accompanied by a viral infection.
Presentation is with sudden onset of pain in the hip or a limp. There is no pain at rest, but there is decreased range of movement, particularly external rotation. The pain may be referred to the knee. The child is afebrile or has a mild fever and does not appear ill.

158
Q

You are asked to review a baby who is 24 hours old. He was born by vaginal delivery following a normal pregnancy. The midwife has noted that the baby’s feet look abnormal. On examination the feet are inverted but appear to be of a normal size. On passive movement of the foot you are able to manipulate the joint so that the upper surface of the foot can touch the shin.

What is the most likely diagnosis from the list given below?

Positional talipes equinovarus
Tarsal coalition
Talipes calcaneovalgus
Talipes equinovarus
Vertical talus
A

This is a common problem and is caused by intrauterine compression. The foot is of normal size, the deformity is mild and can be corrected to the neutral position with passive manipulation.

Tarsal coalition is a diagnosis made in adolescent years.

159
Q

Harry is an 18-month-old baby. He is brought to the Accident and Emergency department as he has stopped walking, and his mother is worried his left leg is painful. On examination he will not weight-bear, and cries when his left leg is moved passively. He is afebrile and the rest of his examination is normal, except that you feel his sclera have a slight blue tinge. An X-ray is performed, which shows a fracture of his femur, but there is also a comment that the bones look osteoporotic. On further questioning from his mother, she cannot give any history of trauma or any incident that could have caused the fracture. On looking back through the Accident and Emergency notes you note that he broke his left arm 4 months ago and again his mother could not give any explanation to why this had happened. Harry’s older sister is with them and has her arm in a plaster, having recently fractured it.

What is the most likely diagnosis for Harry?
Unwitnessed accidental injury
Non-accidental injury
Osteogenesis imperfecta
Osteopetrosis
Osteomyelitis
A

Osteogenesis imperfecta

Harry is likely to have osteogenesis imperfecta (type 1). It presents with fractures in early childhood and the bones look classically osteoporotic on X-ray. The children often have blue sclera but this can be difficult to identify with confidence. The condition is autosominal dominant and it is likely his older sister Amy has it as well.

160
Q

Annette is a 14-year-old mixed-race girl who attends the paediatric outpatients. She complains of worsening occipital headaches that now happen daily. They occur mainly in the mornings, and sometimes wake her from sleep. Her mother says she is doing less well at school than previously, and has become a difficult and grumpy teenager. She sometimes vomits in the mornings. She has no other medical problems though she is on the oral contraceptive pill which was prescribed by her general practitioner.

Select the most likely diagnosis.

Migraine
Tension headache
Medication side-effect
Raised intracranial pressure due to a space-occupying lesion
Idiopathic intracranial hypertension (benign intracranial hypertension).

A

Raised intracranial pressure due to a space-occupying lesion

This girl has raised intracranial pressure due to a space-occupying lesion. The headaches are worsening, and occur when lying down and wake her from sleep. Increased intracranial pressure is associated with morning vomiting. There is also behaviour change and worsening educational performance.

Idiopathic intracranial hypertension also presents with features of raised intracranial pressure but is less likely to affect her behaviour and school performance, though they can be difficult to differentiate on clinical history alone.

161
Q

Gerald is a 10-year-old white British boy. He is seen in the special school clinic with his mother, who is just recovering from cataract surgery. He has moderate learning difficulties, and is teased because of his marked facial weakness. He is unable to walk long distances. His mother says that he struggles to release things once he grabs them. He has no other medical problems. He has not had any investigations performed.

Select the most useful diagnostic test from the list.

Serum creatinine kinase
Muscle biopsy
DNA testing for trinucleotide repeat expansion of myotonic dystrophy
Nerve conduction studies
Electromyography
A

DNA testing for trinucleotide repeat expansion of myotonic dystrophy

The most useful diagnostic test here would be DNA testing for trinucelotide repeat expansion of myotonic dystrophy. The clinical history suggests myotonic dystrophy because of his learning difficulties, myotonia, facial weakness and limb weakness. It was dominantly inherited from his mother, who has cataracts as a result.

162
Q

Olive is a 5-year-old white British girl who had a myelomeningocele repaired shortly after birth. She is a normally cheerful girl who uses a wheelchair for mobility. She attends the paediatric ward because she has a 4-day history of fever, lethargy, vomiting and abdominal pain. She normally opens her bowels twice a week and does intermittent urinary catheterisation 3 times a day with the help of her mother, but despite this she is dribbling urine.

Select the most likely complication that is causing her new symptoms.

Hypertension and renal failure
Hydrocephalus
Tethering of the spinal cord
Urinary tract infection (UTI)
Constipation
A

UTI

Olive had a myelomeningocele that has caused a neurogenic bladder (she has to use catheterisation to maintain continence) and bowel (she is normally constipated). A neurogenic bladder predisposes to developing a UTI, due to the stagnant urine lying in the bladder. Her symptoms of fever, abdominal pain, vomiting and urinary incontinence are most likely to be due to a UTI.

163
Q

Angelo, a 15-month-old mixed-race boy, has been unwell with a runny nose and cough for a day when his father brings him to the Accident and Emergency department. At lunch he suddenly became stiff, his eyes rolled upwards and both his arms and legs started jerking for 2 minutes. He felt very hot at the time. When examined 2 hours later he has recovered fully. This is the first time this has happened. He has a normal neurological examination and is acquiring his developmental milestones normally. He has no other medical problems. The triage nurse performed a blood glucose which was 4.2 mmol/L.

What would be the most appropriate investigation?

Electroencephalography (EEG)
Computed Tomography (CT) scan of the brain
Magnetic Resonance Imaging (MRI) of the brain
Electrocardiograph (ECG)
No investigation required

A

No investigation required

Angelo had a simple febrile seizure, secondary to a respiratory tract infection. A febrile convulsion is a clinical diagnosis and does require investigation.

164
Q

Pamela is an 8-year-old Indian girl with recurrent seizures. She is reviewed in the epilepsy clinic. She has 3 or 4 seizures a month, where she lets out a cry, her arms and legs become stiff, her eyes roll upwards, and then she jerks her arms and legs. This lasts about 3 minutes. Afterwards she sleeps for 2 hours, and is then back to normal. She is doing well at school but is sometimes missing attending because of her seizures. She is currently not on any medication and has no other medical problems.

What would be the best intervention for this child?

Ketogenic diet
Vagal nerve stimulation
Anti-epileptic drug (AED) therapy
Home schooling
No intervention required
A

Anti-epileptic drug (AED) therapy

This child should be started on an anti-epileptic drug (AED) such as sodium valproate. Pamela has recurrent generalised tonic-clonic seizures, which is affecting her quality of life as she is missing school.

165
Q

Sharon is an 11-year-old Chinese girl who attends the paediatric assessment unit. She has had occasional headaches for the last 3 months. Today was her first day at secondary school. During maths class she developed her usual throbbing left-sided headache, associated with nausea. Over the next hour, she lost her temporal vision in her right eye, and found she only had minimal movement in her right arm. The left side of her mouth was drooping. Her parents were called, who rushed her to hospital. She is now feeling much better, has normal vision and can move her arm, although she has some residual weakness of her mouth. She has no significant past medical history except some episodes of abdominal pain as a younger child. Her mother tells you that she also suffers from headaches.

Select the most likely diagnosis.

Migraine
Tension headache
Subarachnoid haemorrhage
Raised intracranial pressure due to a space-occupying lesion
Idiopathic intracranial hypertension
A

Migraine

The headache is typical of a migraine, being unilateral and throbbing, and associated with nausea. This is a complicated migraine because it is associated with neurological phenomena (monoplegia, hemianopia).

166
Q

Aparna is a 2-year-old Indian girl who lives in the UK. She presents to her general practitioner as she has been unsteady on her feet for a day, having had diarrhoea during the previous week. On examination she is afebrile with reduced muscle power and tone, and no tendon reflexes can be elicited in her lower limbs. She is referred urgently to the paediatric hospital and 6 hours later she is unable to stand and the tendon reflexes in her upper limbs are now absent. She has no other medical problems and has been fully immunised.

What is the most likely diagnosis?

Guillain–Barré syndrome
Myasthenia gravis
Muscular dystrophy
Poliomyelitis
Spinal muscular atrophy
A

GBS

Guillain–Barré syndrome (post-infectious polyneuropathy). The classical presentation is with ascending weakness. She must be closely monitored, as a serious complication is respiratory failure.

167
Q

Which of these Western European countries has the highest incidence of teenage pregnancies?

France
Portugal
Norway
United Kingdom
Holland
A

UK

Becoming a teenage mother may be a positive life-choice, especially if there is considerable support from extended family. However, when the pregnancy is unintended, there can be many adverse consequences for mother and child, especially if she is unsupported or living in poverty.

168
Q

Claire is 14 years old. She visits her general practitioner as she wants to go on the oral contraceptive pill. She is sexually active and has a boyfriend with whom she has been in a relationship for 6 months. Her mother is not aware she has come to see the doctor today and is also not aware she is sexually active. Claire does not want her mother knowing she is getting the oral contraceptive pill. She has no other medical problems. She has regular periods and her blood pressure is normal.

What is the recommended advice to give her?

You will only give her the pill if her parents are present to consent in writing
You will prescribe the pill and promise not to tell her parents
You will prescribe the pill and encourage her to tell her mother she is going to start the pill
You will prescribe the pill but will inform her parents you are doing so
She cannot have the pill as she is not legally allowed to have sex

A

You will prescribe the pill and encourage her to tell her mother she is going to start the pill

It is usually desirable for the parents to be informed and involved in contraception management. She should be encouraged to tell them or allow the doctor to, but if the young person is competent to make these decisions for herself, in the UK the courts have supported medical management of these situations without parental knowledge.

169
Q

Which of the following is the most common cause of death in adolescence in the UK?

Cancer
Heart disease
Infection
Injury and poisoning
Neurological disease
A

Injury and poisoning

In the UK injury and poisoning account for 60% of deaths in 15–19-year-olds. Alcohol is thought to be a contributing factor in a third of those deaths.

170
Q

Olivia is a 16-year-old girl who presents to the local pharmacy for ‘emergency contraception’. She has had unprotected sex with her boyfriend, who is also 16 years old. She has no other medical problems and is not currently on any medications.

How long after unprotected intercourse can emergency contraception be used?

12 hours
24 hours
48 hours
72 hours
1 week
A

72hrs

Emergency contraception is available from a pharmacist without prescription for those aged 16 years and over, and on prescription for those under 16 years. If taken within 72 hours, it has a 2% failure rate.